Sports Study Questions

Réussis tes devoirs et examens dès maintenant avec Quizwiz!

Unlike hepatitis B and Hepatitis C, Hepatitis A is unique in that this virus? (A) Is a common bloodborne pathogen (B) It is usually spread by the fecal-oral route (C) Has no available vaccine (D) Can be transmitted percutaneously (E) Results in a chronic carrier state

B - While there have been documented cases of transmission of Hepatitis B & C through sports, no such transmission of Hepatitis A has been found. Transmission of hepatitis A is related to contaminated food or drink. Hepatitis B & C are bloodborne pathogens, not Hepatitis A. Hepatitis C has no available vaccine currently, Hepatitis B & A both have vaccines. Hepatitis A, B & C can all be transmitted percutaneously. Hepatitis B & C can result in carrier states, Hepatitis A has not been associated with carrier state.

Which of the following is an abnormal ECG finding in a trained athlete? (A) sinus bradycardia (B) first degree AV block (C) left atrial enlargement (D) incomplete RBBB

C - A is not correct as sinus bradycardia is almost universal in athletes due to a physiological adaptive change of the autonomic nervous system. B is not correct as around 35% of athletes have this ECG change due to AV conduction slowing and block are mediated by increased parasympathetic tone. Resolution with exercise excludes a pathologic process. C is the correct answer as left atrial enlargement can be due to structural and/or electrical abnormalities. D is not correct as incomplete RBB is a QRS duration <120 ms and can be seen in 35-50% of athletes. The right ventricle conduction delay is not within the His-Purkinje system, but is caused by the enlarged right ventricle cavity size or increased cardiac muscle mass

A healthy adolescent female softball player presents to your office after several weeks of increasing pain just distal to the ankle joint. She reports no trauma. She has painful nearly flat feet. Her mother reported a similar problem in her youth. With a normal exam otherwise, what do you suspect? (A) Navicular fracture (B) Lisfranc fracture (C) Referred pain - mid-shaft metatarsal fracture (D) Referred pain - osteochondral defect - tarsal (E) Tarsal coalition

E - The tarsal coalition is more common in adolescents, has a familial risk and occurs in both sexes equally. The presentation for tarsal coalition is as described while the other diagnoses listed would be either traumatic or demonstrate different exam findings.

While numerous laboratory studies have searched for neuroprotective treatment approaches to traumatic brain injury, no therapies have successfully translated from the bench to the bedside. Concussion is a unique form of brain injury, in that the current mainstay of treatment focuses on both physical and cognitive rest. Treatments for concussion are lacking. Current research in rats have shown which of the following treatments reduce injury post-head trauma in rats? (A) significantly increasing Omega-3 polyunsaturated fatty acids in the diet (B) short term use of vasodilators in the immediate post-concussive phase (C) increased oral glucose intake (D) increased sodium intake

With respect to neuroprotection in the context of improving outcomes following TBI, multiple preclinical studies have suggested that DHA and/or EPA supplementation may have potential benefit through a multitude of diverse, but complementary mechanisms. Studies utilizing rodent models of experimental injury have shown that pre-injury dietary supplementation with fish oil effectively reduces post-traumatic elevations in protein oxidation resulting in stabilization of multiple molecular mediators of learning, memory, cellular energy homeostasis and mitochondrial calcium homeostasis as well as improving cognitive performance. The benefits of pre-traumatic DHA supplementation have not only been independently confirmed, but DHA supplementation has been shown to significantly reduce the number of swollen, disconnected and injured axons when administered following traumatic brain injury in rodent models. Vasodilators would increase the risk for a reversal of the reduced blood supply to the brain that prevents cerebral edema. Increased glucose has not shown to help. Increased sodium has not shown to help with this.

A 28-year-old male, right hand dominant Olympic weightlifter presents with unilateral pain and swelling in his right upper extremity. The pain and swelling is worse with weight lifting and strenuous exertion of his right arm and improves with rest and elevation. On exam, the patient has prominent superficial veins visible over the right subclavian area and a palpable cord over the basilic and axillary veins. The right upper extremity has non-pitting edema with slight erythema. Pulses are normal and Adson's test negative. There is no muscle atrophy. Tinel and Phalen's signs were negative. Spurlings test is negative and the rest of the neurological exam is normal. The left upper extremity shows no abnormalities. What is the initial imaging modality of choice? (A) Ultrasound (B) MRI (C) CT Scan (D) EMG/NCS

A - A is the correct answer. Paget Schroetter, or effort induced thrombosis, is a spontaneous thrombosis of the subclavian vein usually present in otherwise healthy young individuals. This syndrome often occurs in physically active individuals after unusually strenuous use of the arm and shoulder. Ultrasound is the initial test for diagnosis and to rule out DVT. Work up should include hypercoaguability testing. Treatment is surgical decompression and anticoagulation. However, thoracic outlet syndrome can mimic these symptoms which also may cause an arterial obstruction or nerve entrapment. Neither MRI or CT scan are helpful in initial diagnosis of either syndrome. Although nerve entrapment may be part of the entity of thoracic outlet syndrome, EMG/NCS likely will not change initial conservative management. Ultrasound will help rule out a possible serious condition.

A football player presents to you following practice with an injury to his right 5th finger. On exam, he is unable to extend at the PIP joint. You are concerned about this injury developing a boutonnière deformity. Which of the following correctly describes the mechanism causing this deformity? (A) Rupture of the extensor central slip of the PIP with subsequent contraction of the lateral bands leading to extension deformity of the DIP (B) Avulsion of the flexor digitorum profundus tendon at the DIP (C) Disruption of the extensor mechanism at the DIP (D) Extensor tendon subluxation at the MCP joint (E) Rupture of the extensor hood of the MCP joint

A - A is the correct answer. Tendon injuries of the hand can lead to multiple deformities and issues. A boutonnière deformity occurs commonly following an injury to the central slip, which connects the extensor tendon to the middle phalanx at the PIP joint. This commonly occurs with a forced flexion of the PIP joint against resistance. If unrecognized, this can then lead to a contraction of the lateral bands of the PIP joint causing a flexion deformity of the PIP and an extension deformity of the DIP. Proper treatment with the PIP in extension for 6 weeks can prevent this deformity. Answer B is the common mechanism for a Jersey finger leading to inability to actively flex the DIP joint. Answer C is the common mechanism for a Mallet finger leading to inability of active extension of the DIP joint. Answer D is common following trauma to the MCP and leads to inability of extension at the MCP joint. Answer E is commonly referred to as

The starting wide receiver on your D1 college football team comes to see you for decreasing performance with military press during his in-season weight-lifting program. He has been playing well this year, but has noticed some difficulty with overhead activities with his right shoulder. He denies shoulder pain and cannot remember a particular injury. His right shoulder exam is remarkable for the following: atrophy of his supraspinatus muscle on the right. No tenderness to palpation. He has a 4/5 empty can test and external rotation. He also has a positive load and shift test on the right. He has a normal exam of the left shoulder. What would you expect to find on an MRI of his right shoulder? (A) A cyst compressing the suprascapular nerve in the scapular notch (B) A partial thickness articular sided supraspinatus muscle tear (C) A partial thickness bursal supraspinatus tear (D) Fatty replacement in the supraspinatus muscle

A - Answer A is correct. He likely has a labral tear which has lead to a cyst formation compressing the suprascapular nerve in the scapular notch leading to atrophy of the supraspinatus muscle (Answer A is correct). Partial thickness tears are usually seen in overhead throwers. You should not see suprascapular atrophy on the physical exam with partial tears (Answers B and C are incorrect). Fatty replacement is a late finding on MRI with untreated full thickness tears. This is usually found in an older patient population (Answer D is incorrect).

A diver experiences ear pain, ear fullness, and tinnitus while descending during SCUBA diving. After rising, the most appropriate treatment for his symptoms include which of the following? (A) Oral psuedoephedrine (B) ENT referral (C) Hyperbaric oxygen (D) Intravenous Fluids (E) Antibiotics

A - Explanation: this diver is experiencing middle ear squeeze, commonly associated with excess pressure in the middle ear, associated with diving and concurrent (usually) middle ear, nasal, and sinus congestion. In addition to removing the offending pressure, a decongestant and rest from diving are the treatments of choice. Antibiotics would only be indicated if there was a concern for an infection (ex. with purulent drainage). Should he have disruption of the tympanic membrane, which can occur with middle ear squeeze, referral to ENT would be appropriate. ENT is also appropriate for inner ear squeeze, in which the Valsalva on descent was over vigorous and the round window was damaged. Hyperbaric oxygen is the treatment for disorders that commonly occur on ascent from a dive, particularly air embolism and decompression sickness. IV fluids can also be helpful in displacing some of the added air bubbles. Pneumothoraces and pneumomediastinum may resolve spontaneously, if small, or may require thoracostomy.

A 13-year-old male baseball player complains of worsening right shoulder pain with pitching. He has recently started playing with his high school baseball team in addition to his traveling team. On examination, he complains of pain with palpation to the lateral aspect of his shoulder. The most likely diagnosis for this patient is? (A) Little Leaguer's Shoulder (B) Biceps tendon strain (C) Supraspinatus tear (D) Anterior shoulder impingement (E) Glenohumeral labral tear

A - Little Leasguer's shoulder or proximal humeral epiphysiolyisis is an injury to the proximal humeral growth plate that effects baseball players (more often pitchers) between the ages of 11- 14 years old. This injury represents a Salter-Harris type I fracture of the proximal humerus. It is thought to be caused by the rotational stress on the growth plate from high speed throwing. Athletes generally only have symptoms of shoulder pain during throwing, but complain of pain with pressure to the lateral aspect of the proximal humerus. Radiographs nearly always show widening of the proximal humeral epiphysis, and sometime also show lateral metaphyseal fraying, demineralization, or sclerosis. Primary treatment for proximal humeral epiphysiolysis consists of rest from throwing, on average for three months. Prevention of this injury is achieved by early focus on proper throwing mechanics, delay of throwing curveballs or sliders until the age of 13, limiting pitch counts based on age, and taking a break from baseball every year. Based on the patient's age, sport, symptoms, and examination, little leaguer's shoulder is the most likely diagnosis of the choices given. Biceps tendon strain generally causes pain at the anterior aspect of the shoulder, over the bicipital groove, and pain with resisted biceps contraction. A rotator cuff muscle tear more frequently occurs in the older athlete population. A supraspinatus tear specifically would cause difficulty with resisted shoulder elevation in a scapular plane, either with the shoulder externally or internally rotated. Shoulder impingement occurs due to structural narrowing under the anterior acromium, causing compression of the underlying muscles; the patient will demonstrate positive Neer Impingement and Hawkins signs. Glenohumeral labral tears can occur in young overhead throwers, but usually the athlete will experience a deeper shoulder pain and additional symptoms, such as popping, decreased range of motion, apprehension, and weakness of the shoulder.

A 28-year-old right hand dominant male presents with left shoulder pain for two days. He denies any trauma and states his daughter fell asleep laying directly on his shoulder for 3 hours the night before the the pain started. Exam including range of motion and strength in the shoulder/upper extremity is normal, and he is treated conservatively. He follows up in 6 weeks and states the pain has resolved after 1 week, but now has weakness with his left arm. On exam, you note visible atrophy of the supraspinatus and infraspinatus muscles and his strength is only 3 out of 5 with resisted external rotation and empty can testing. There is also scapular winging on serratus testing. All other strength and range of motion testing is normal. MRI of cervical spine and shoulder is normal except for noted supraspinatus and infraspinatus atrophy. EMG shows suprascapular nerve and long thoracic nerve involvement. What is your diagnosis? (A) Parsonage Turner Syndrome (B) Thoracic Outlet Syndrome (C) Paget Schroetter Syndrome (D) Cervical Foraminal Stenosis

A - Parsonage Turner Syndrome/acute brachial neuritis is a brachial plexopathy/brachial neuritis consisting of a complex constellation of symptoms with abrupt onset of shoulder pain, usually unilaterally, followed by progressive neurologic deficits of motor weakness, dysesthesias, and numbness. The findings on EMG can be patchy, in contrast to a radiculopathy. The long thoracic nerve and anterior interosseus nerve are reported to be involved more frequently than other nerves. The pathophysiology is not well understood, but is thought to potentially be associated with recent viral illness or autoimmune process. Treatment consists of NSAIDs or narcotic pain medicines in the acute period for pain control, range of motion exercises and isolated strengthening of the shoulder girdle. Reinnervation of the affected muscles typically takes place over several months, but can take up to 2 years. Thoracic outlet syndrome (TOS) would not create muscle atrophy or findings on EMG listed above (B is incorrect). True neurogenic TOS is quite rare, but when present tends to involve the lower trunk of the brachial plexus (hand intrinsics). Paget Schroetter Syndrome is a complication of vascular thoracic outlet syndrome that involves deep venous thrombosis of the xillary/subclavian vein (C is incorrect). Cervical foraminal stenosis would show findings on the cervical MRI and would be expected to have weakness throughout a myotomal distribution, rather than the sometimes patchy distribution of Parsonage Turner Syndrome (D is incorrect).

A 26 year old professional soccer player sustained a grade 3 hamstring strain during a practice. He was unable to finish the practice and had to be helped off the field. He was immediately assessed by the athletic trainer who made the diagnosis and subsequently referred him to the team physician for further management. Physical exam and MRI findings confirmed the diagnosis. The player is concerned about the length of time he will be unable to play. The most appropriate approach to rehabilitation for this player would be which of the following: (A) Pain control; motion; strength; neuromuscular control; return to play (B) Pain control; strength; motion; neuromuscular control; return to play (C) Pain control; neuromuscular control; strength; motion; return to play (D) Motion; pain control; neuromuscular control; strength; return to play

A - Principles of an appropriate rehabilitation schedule start with pain control followed by motion, strength, and neuromuscular control and then return to play as tolerated. This is known as the baseball diamond approach to rehabilitation. You must complete one base before moving onto the next. Pain control is achieved with PRICE protect/rest/ice/compression/elevation); medications including acetaminophen/NSAIDS/tramadol or opioids (NO STEROIDS); other modalities including electrical stimulation or ultrasound (after initial period of inflammation). Restoration of range of motion (ROM) helps with lessening edema and promotes tissue healing. ROM is also a prerequisite for restoration of strength/endurance. Generally begins with active assisted (AAROM) followed by active (AROM) and finally resisted (RROM). Once ROM is restored the athlete can progress to the strengthening phase. The strengthening phase is important for restoration of functional strength and endurance. There are three different types of strengthening: isometric (no joint movement), isotonic (change in muscle length with constant resistance but variation of speed), and isokinetic (variable resistance with constant speed). Finally, restoration of neuromuscular control/proprioception is achieved before return to play. Coordination requires an afferent (proprioceptive) input detection and processing to produce an efferent (muscle contraction) output. Detect deficits with assessment of balance, movement patterns, and sport specific scenarios. Neuromuscular control training utilizes equipment such as wobble boards, foam rollers, body blades, and even plyometric training. The athlete can then progress through a graduated return to play (RTP) with continued appropriate monitoring and post RPT rehabilitation.

Creatine is a commonly used supplement. The body excretes approximately two grams per day and replaces 50% by diet and 50% is made intracelluarly. Supplementing with exogenous creatine is thought to maximize the stored level in the cells. This helps with repeated short bouts of high-intensity physical activity by? (A) Increasing phosphocreatine reserves for creatine kinase (B) Binding to troponin allowing rapid contraction of muscles (C) Acting as a rapid fuel source in muscles forming glycine and arginine (D) Increasing the water content in the muscle cells maintaining hydration (E) Acting as a co-factor for the enzyme pyruvate dehydrogenase

A - Supplementing with creatine increases phosphocreatine reserves in the muscle cells. Creatine is an amino acid synthesized from the amino acids glycine, arginine, and methionine which is metabolized in the liver, kidneys, and pancreas when dietary ingestion of creatine is not enough to meet bodily needs. Creatine binds with phosphate to form phosphocreatine (PCr). Creatine and PCr storage allows regeneration of ATP from ADP for energy demands from the creatine kinase reaction. It does not bind to the muscle proteins nor acts as a co-factor for PDH. Creatine supplementation may decrease the risk of dehydration during exercise by increasing total body water, but this is not how it may help with highintensity activity.

A 54-year-old female presents to your office with complaints of left shoulder pain. The pain has been going on for a couple of months and it hurts with motion in any plane. There was no injury to the shoulder and she had never experienced pain like this in the past. On examination, she has a limitation of passive range of motion in external rotation and abduction. You make a diagnosis of adhesive capsulitis. Which of the following are true regarding adhesive capsulitis? (A) There is a predilection for adhesive capsulitis in patients with diabetes mellitus (B) Treatment with oral corticosteroids shorten the course of the condition (C) Intraarticular shoulder injection with corticosteroid is less effective than oral corticosteroids (D) Surgical intervention is not helpful in the treatment of adhesive capsulitis

A - There is a higher incidence of adhesive capsulitis in patients with insulin dependent and non-insulin dependent diabetes. Oral corticosteroids have been show to provide short term pain relief but have not been proven to decrease the duration of the condition. Intraarticular corticosteroids have been shown to be superior to oral corticosteroids in regards to short term pain relief and increased range of motion. Surgical intervention plays a role in those patients that have not received adequate response from conservative treatments. Manipulation under anesthesia and capsular relief have both been shown to be helpful with pain relief and recovery.

A 17-year-old high school football player tackles an opposing player on the other side of the field. Your view was somewhat obstructed, but he appeared to hit with his head down. He comes off the field complaining of a tingling sensation in both hands. He denies any neck pain and appears to have full grip strength. What is your next action? (A) Place him a cervical collar and on a spine board immediately (B) Evaluate for cervical spine tenderness, and if positive, place him in a cervical collar and on a spine board immediately (C) After ensuring there is no cervical spine tenderness, evaluate cervical range of motion, and if painful, place him in a cervical collar and on a spine board immediately (D) Evaluate cervical range of motion and upper extremity strength and sensation, and if normal have him sit out the rest of the game, but may resume practices next week (E) Evaluate cervical range of motion and upper extremity strength and sensation, and if normal, allow him back in the game

A - This athlete is describing the so called burning hands syndrome, in which the athlete has painful paresthesias in both hands. This pattern is suggestive of central cord syndrome, and is one variant of transient quadriparesis (TQ). By definition, TQ is a generally self limited neurological episode, but symptoms can last up to 36 or 48 hours in some cases. Athletes will present with sensory symptoms, with or without motor symptoms, in more than one limb. Sensory complaints may ranage from paresthesias to completely absent sensation. Motor function may be completely absent, some weakness, or normal with only sensory symptoms. Any athlete with neurological symptoms in more than one limb should be suspected of having TQ and treated with full cervical spine precautions to prevent potential progression of the injury. For this reason, A is correct. B is incorrect because athletes with TQ generally do not complain of neck pain or tenderness, and therefore the diagnosis may be missed if this is the only criteria used. They may complain of a burning sensation on the back of the neck though. C, D, E are all incorrect because in any athlete with neurological symptoms in more than one limb, full cervical spine precautions should be taken immediately to prevent the possibility of further progression of the injury.

An elderly male presents complaining of medial knee pain in the absence of trauma. His pain is associated with morning stiffness and has been present for the past few weeks. He does see some improvement with ibuprofen. Which of the following is appropriate management in the work-up and/or treatment of arthritis in this patient? (A) Weight-bearing radiographic views may assist in determining loss of joint space (B) Ibuprofen is the first drug of choice in this case (C) Glucosamine is likely to worsen pain in this patient and therefore should not be recommended (D) Arthroscopic debridement in the treatment of osteoarthritis is effective

A -The Answer is A For evaluation of suspected OA, order weight-bearing anterior views. Acetaminophen is the initial drug of choice for most patients because of its good efficacy and greater safety than NSAIDs. Glucosamine has been shown to be effective in reducing pain, improving function, and possibly slowing the progression of OA. Arthroscopy and arthroscopic debridement for treatment of OA is not effective.

You are the sideline team physician for your local college soccer team. You are working at a soccer game, when a player falls on an outstretched hand. She is taken out of the game and you evaluate her shoulder, determining an anterior shoulder dislocation is present. Which of the following is true about bony abnormalities following an anterior shoulder dislocation? (A) Bony bankart lesions are present in 90% of anterior shoulder dislocations (B) Hill-Sachs lesions may lead to increased destabilization of the glenohumeral joint and create an increased risk for recurrent dislocation (C) Hill-Sachs lesions are rare. These lesions are seen less than 5% of the time after an anterior shoulder dislocation (D) Bony bankart and bankart lesions are the same injury, but health care providers use the names interchangeably

B - A is incorrect because bony bankart lesions are present in about 5% of shoulder dislocations. Soft tissue bankart lesions are much more common. B is correct. C is incorrect because Hill-Sachs lesions are present in 35-40% of shoulder dislocations and in up to 80% of recurrent shoulder dislocations. Bony bankart and bankart lesions represent two different injuries. A bony bankart is an avulsion fracture of the glenoid. A bankart lesion is an avulsion of the antero-inferior glenoid labrum at its attachment to the inferior glenohumeral ligament complex

A 16-year-old male snowboarder had an accident during the Olympic competition. It was significant enough that it was decided to transport him to the hospital. En route, he complained of left shoulder pain, but remained hemodynamically stable during transport. At the hospital, his hemoglobin remained normal and stable throughout. CT scanning with contrast revealed a Grade II Splenic injury. Which of the following are correct regarding his initial evaluation, management and disposition? (A) The spleen is rarely injured during sport (B) Non-operative management would be preferred (C) Splenic rupture is of minor concern in this patient (D) Ultrasound is the preferred method of imaging in stable patients (E) He should be vaccinated immediately

B - Answer A is not correct as the spleen is the most commonly injured abdominal organ in sports. Answer B is the best answer for several reasons. He is an adolescent who is hemodynamically stable. Preservation of the spleen is always preferable in the long term. And since his competitive season is likely over for a while after the Olympics, he would be an excellent candidate for non-operative management which is the currently preferred method of management. Healing can take several months. Answer C is incorrect because delayed splenic rupture is the greatest concern after 48 hours in the non-operatively managed patient. Answer D is incorrect because CT scanning has been shown to be superior to US. If the patient is unstable, portable ultrasound would then be the preferable imaging method, but is not pertinent in this scenario. Answer E is correct if the patient is thought to need an urgent splenectomy. There was no indication that emergent surgery was needed, and therefore is not the best answer

A 19-year-old female soccer player is involved in a collision with another player. She had her neck flexed, and hit the frontal aspect of her head into the knee of the other player. Currently she is lying on the ground, unresponsive and not breathing. Strong pulses are noted. Her pupils are unequal. What is the most appropriate next step in management while awaiting EMS arrival? (A) Do not wait for EMS and transport the player to the Emergency Department as quickly as possible for CT scan (B) Use jaw-thrust maneuver to open the airway and ventilate using bag valve mask (C) Place a nasopharyngeal airway and ventilate using bag valve mask (D) Use head-tilt-chin-lift to open airway and ventilate using bag valve mask

B - B is the correct airway maneuver, as the jaw-thrust technique will help prevent against hyperextension of the neck on a patient with possible spinal cord injury. This should also be used in conjunction with a cervical collar and manual stabilization of the neck. D is incorrect because the head-tilt-chin-lift uses neck extension, which should not be performed given the potential for cervical spine injury in this patient. C is incorrect as the use of a nasopharyngeal airway in a closed head injured patient may result in iatrogenic intracranial placement of the device, especially if there is a basilar skull fracture. An oropharyngeal airway may be placed, or the patient may be orotracheally intubated. A is incorrect because while this patient needs quick transport to the Emergency Department for evaluation of head injury, there is also possibility of cervical spine injury. This patient should not be moved from her current position until placed in the proper precautions, which should be maintained during transport. This will include a cervical collar and backboard. She also needs immediate airway assistance given she is not breathing.

improved but he continues to have pain and tightness in his thigh. Radiographs demonstrate a small area of heterotopic bone likely within the rectus femoris. His initial treatment involved icing the area for 15 minutes after the game. What is the recommended treatment option? (A) Surgery to excises the heterotopic bone (B) Physical therapy for stretching, strengthening and modalities. Monitor for 12 months. If symptoms persist and heterotopic bone is stable, then consider surgical excision of the heterotopic bone (C) Prednisone taper over the next 2 weeks (D) Deep tissue massage

B - B is the correct answer. Myositis ossificans is the formation of heterotopic bone deposition that has been reported to be a sequala of 9-17% of muscle contusions. It can be unstable and continue to grow for up to 1 year. Surgery prior to stabilization of heterotopic bone formation can lead to recurrence. Prior to surgery, a bone scan should no longer show increased uptake in the injured area. NSAIDs such as indomethacin at the initial time of injury have been theorized to prevent myositis ossificans but nothing to suggest it modifies hetorotopic bone maturation after heterotopic bone has formed. NSAIDs can be used for pain relief. Corticosteroids have been shown to cause muscle atrophy. Physical therapy can assist with return to function and symptom management. If the area involved is asymptomatic, then there is no need for surgical intervention.

A 68-year-old right hand dominant female with multiple medical problems presents to your clinic with chronic, aching right shoulder pain for the past few months. She denies any previous injury, surgery or any night time symptoms. On examination she is only able to forward flex her right shoulder to about 80 degrees, both actively and passively. She is also noted to have limited shoulder abduction and external and internal rotation. Shoulder radiographs are normal. You start her on anti-inflammtory medication, teach her Codmans exercises to do at home, and schedule followup in 4 weeks. Which of the following medical problems is most commonly associated with her diagnosis? (A) Morbid obesity (B) Diabetes mellitus (C) Ankylosing spondylitis (D) Congestive heart failure

B - B is the correct answer. This patient has adhesive capsulitis, also commonly referred to as a 'frozen shoulder'. It most common in women between 40-70 years old. Answer B is correct because adhesive capsulitis is common in persons with insulin-dependent and non-insulin-dependent diabetes and in those with prediabetes. It can also be seen in patients with thyroid disease. Recent chest surgery and neurological conditions such as stroke are also predisposing factors. Persons with a history of adhesive capsulitis are at increased risk of developing the condition on the contralateral side. Recurrence on the affected side is also possible, especially in patients with diabetes, though it is not common. The hallmark of adhesive capsulitis is decreased active and passive range of motion and shoulder pain. Answers A, C and D are incorrect as although it may be possible for adhesive capsulitis to be seen in these conditions, it is not as frequent as with diabetes.

A 7-year-old male presents to your office with an eight month history of right hip and knee pain. Neither the parents or the child recall any trauma. Upon examination, he has an antalgic gait. His hip joint is not erythematous and there is no tenderness to palpation. He has limited range of motion in the affected hip and atrophy of the proximal thigh muscles. His temperature is 36.9 degrees C (98.5 degrees F). Radiographs of the knee and hip (including an AP and frog-leg view) were ordered and remarkable for collapse of the femoral head. What is the appropriate treatment? (A) Immediate surgical screw fixation (B) Maintain containment of the femoral head within the acetabulum (C) Ultrasound of hip (D) Needle aspiration of the joint

B - B is the correct answer. This patient likely has Legg-Calvé-Perthes disease, an avascular necrosis of the femoral head. Typically, patients are between the ages of 5-7 years old. Symptoms may include pain in the knee, groin or anterior thigh. AP and frog leg lateral radiographs can be used to confirm the diagnosis. The main goal of treatment in this condition is to maintain range of motion and keep the femoral head contained in the acetabulum. A is incorrect as this would be the correct treatment for slipped capital femoral epiphysis (SCFE). In this condition, the capital femoral epiphysis displaces from the femoral neck. It occurs more commonly in obese adolescents. C is incorrect as an ultrasound would not be useful at this time. D is incorrect as the history and exam findings do not suggest infection.

22-year-old male presents to the urgent care clinic with complaint of left wrist and elbow pain after he slipped on some ice 2 hours ago and fell on his outstretched hand. You decide to obtain radiographs of his left arm. Which of the following findings would be consistent with a Monteggia fracture? (A) Fracture of distal third of ulna with anterior dislocation of radial head (B) Fracture of proximal third of ulna with anterior dislocation of radial head (C) Fracture of proximal third of ulna with posterior dislocation of radial head (D) Fracture of distal third of ulna with posterior dislocation of radial head

B - Correct answer is B. Monteggia's classic description is a fracture of the proximal third of the ulna with anterior dislocation of the radial head. This fracture-dislocation (Type 1) is the most common form of -78%). Answers A, C and D are incorrect. The most common mechanism of injury is a fall on the outstretched, extended and pronated elbow. A direct blow to the posterior aspect of the neurapraxia of the radial nerve, particularly the posterior interosseous nerve. Prognosis for spontaneous recovery is excellent.

Which of the following supplements is appropriately paired with its physiological benefit as an ergogenic aide? (A) Creatine and improved performance in physical activties longer than 90 seconds (B) Carbohydrate - electrolyte beverage and increased energy, decreased faitgue (C) Amino acids and increased muscle mass (D) Vitamin E and decreased exercise induced muscle damage (E) Caffeine and increased muscle mass

B - Creatine has been shown to improved muscle energy availability in muscles, but only for short bursts of activity, less than 30 seconds. Therefore, A is incorrect. Amino acids are thought to increase human growth hormone levels, but there is no clear evidence that this occurs. Therefore, C is wrong. Vitamin E is an antioxidant, but there is not conclusive evidence that it decreases exercise induced muscle damage. Vitamin C has been shown however to decrease exercise induced muscle damage in animal models. Therefore, D is wrong. Caffeine has been shown to mobilize fat and inprove energy sources to active muscles. It has not been shown to inprove muscle mass. therefore, E is wrong.

A 3 phase bone scan can aid in the diagnosis of complex regional pain syndrome. What diagnostic finding would be the most helpful? (A) Focal increased activity on Bone scan (B) Diffuse increased activity with juxta-articular accentuation uptake on delayed images (C) Normal bone scan in late stage of syndrome (D) Focal changes on Phase 1 and 2 (E) Anticholinergics

B - Increased uptake is seen in 2/3 of adult patients with CRPS it is usually diffuse but can be seen on delayed images the best. Focal increased activity is not consistent with CRPS.

Which of the following is not true regarding iron deficiency anemia (IDA) in athletes? (A) Women are at higher risk of developing IDA than men (B) In the initial stage of IDA, serum iron concentrations are low while the ferritin and hemoglobin levels are normal (C) Footstrike hemolysis is a known cause of IDA (D) Inadequate calorie consumption and menstrual losses are common causes in female athletes

B - Iron deficient anemia (IDA) is the most common true anemia found in athletes. Athletes are often asymptomatic but may also have symptoms including weakness, palpitations, shortness of breath, and pica. Women are at a higher risk of developing IDA than men due to menstruation and inadequate dietary intake of iron. Causes of iron loss in athletes include gastrointestinal, genitourinary, sweat and foot strike hemolysis. Gastrointestinal losses are of the greatest importance. Initial lab values in IDA reveal only a low serum ferritin level (answer B is the correct answer because the statement is incorrect). As the anemia progresses, the serum iron and hemoglobin concentrations will also drop.

A 34-year-old carpenter falls three feet and sustains a FOOSH injury (Fall on Outstretched Hand) that results in a carpal ligament instability. Which one of the carpal bones is most commonly involved? (A) Capitate (B) Lunate (C) Scaphoid (D) Triquetrum

B - Lunate is the most common bone involved with instability, i.e. scapholunate disassociation and perilunate instability. The scaphoid is the most commonly fractured carpal bone.

During a pre-participation evaluation of one of your 15 year old patients, the screening questionnaire uncovers that he is attempting weight loss for wrestling season, which starts in two weeks. Which of the following is the best general advice regarding healthy weight loss? (A) Instead of dropping down to a lower weight class, he should try to gain to get into the higher weight class (B) Weight loss should not exceed 1.5% of body weight per week (C) Weight loss should not exceed 0.5% of body weight per week (D) As long as the weight loss is just fat, the rate of loss doesn't matter (E) If he stays above 7% body fat, the rate of loss doesn't matter

B - Many athletes attempt weight loss in pursuit of perceived athletic advantage. If conducted with appropriate care, associated risks can be minimized. Guidelines on this topic suggest that approximately 1.5% of weight can be lost per week without likelihood of negative consequences, such as dehydration or muscle breakdown. Although in some situations, gaining weight may be good advice, it may not be adhered to if it conflicts too strongly with the goals of the athlete. Weight loss of 0.5% per week may be satisfactory, but exceeding 0.5% per week would not usually be unsafe. There is no evidence to support that limiting weight loss to fat only obviates the concerns related to rapid weight loss. Maintaining at least 7% body fat is recommended for male high school athletes, but rapid weight loss can still carry risks even if body fat is above 7%.

A 37 year old male who is otherwise healthy, but minimally active physically has signed up as a charity runner for a local marathon in August. He has been training well per Jeff Galloway's training program for 1st-time marathoners. He comes to see you in June before the race with concerns about hydration for prevention of heat injury. Appropriate recommendations for fluid hydration during endurance events include? (A) Drink at each water stop along the race course (B) Drink ad lib based on thirst (C) Drink an adequate amount of fluids to keep urine output pale (D) Alternate water and glucose-electrolyte solution according to a pre-planned schedule

B - Per a recent position statement on fluid replacement recommendations during marathon running, adequate hydration is very important for peak performance with exercise as well heat stress management. Numerous recent case reports of Exertional Hyponatremia (EH) during endurance events and in particular marathons have prompted changes in recommendations for hydration during these events. Most episodes of hyponatremia are associated with fluid overload or over consumption and therefore many races are even decreasing the number of water stops as well publishing many education flyers on hydration recommendations. The old adage that when you are thirsty you are behind in fluid requirements has fueled this misconception. Drinking fluids ad lib has been demonstrated to be the best protection for fluid overload.

Which commonly prescribed medication is associated with increased risk of osteoporosis? (A) ranitidine (B) valproic acid (C) atorvastatin (D) citalopram

B - Several studies show that anti-epileptics including phenytoin, valproic acid, carbamazepine may lead to excessive bony turnover and demineralization with associated increased risk of osteoporosis

A college sophomore soccer player is diagnosed with acute infectious mononucleosis. Appropriate returnto-play should be based on which of the following? (A) Normal sized spleen by ultrasound (B) 3-4 weeks after onset of symptoms and symptom-free (C) Normal liver function tests (D) Non-palpable spleen on physical exam

B - Splenic rupture is a rare complication of EBV infection, occurring in < 1% of cases. Although rupture usually occurs at rest, physical trauma increases the risk of rupture. Soccer is a contact sport and should be avoided in cases of splenomegaly Physical exam is a poor way to detect splenomegaly. Palpable splenomegaly may be as low as 17% in EBV infection, whereas by US it approaches 100%. Liver function tests do parallel spleen size in mononucleosis but may not normalize at the same time. Even if a spleen has returned to normal size, there is a structural fragility of spleen histologic archtitecture in mononucleosis which is independent of size. Most sports medicine doctors recommend a minimum of 3 weeks rest in mononucleosis.

A 23-year-old male presents to your urgent care clinic in extreme pain. He was playing basketball two hours ago when suddenly he experienced a sharp pain in the groin region. The patient states he has never felt this pain before and that any movement makes his pain worse. He denies sexual activity. His vitals at the time of evaluation: temperature 36.9 degrees C (98.5 degrees F), blood pressure 126/84, pulse 98 and respiratory rate 19. Upon physical exam, the patient's left testicle is extremely tender to palpation. The cremasteric reflex is absent. What is the most appropriate treatment? (A) Immediate surgical consult for potential detorsion and orchiopexy of affected testicle (B) Immediate surgical consult for potential detorsion and orchiopexy of both testicles (C) Admit to the hospital with IV antibiotics (D) Scrotal support, pain control with NSAIDs and ice

B - This patient likely has a testicular torsion. This is indicated by the history of sudden onset during activity and the physical findings (extreme tenderness, absent cremasteric reflex.) An ultrasound could be done to confirm the diagnosis. Treatment for this diagnosis includes immediate surgical exploration and BILATERAL orchiopexy since inadequate gubernacular fixation is usually a bilateral defect, thus making answer #2 correct and choice #1 incorrect. Choice #3 would be most appropriate for epididymitis. The absence of sexual activity and a fever make this less likely. Choice #4 is insufficient treatment in this case. Without surgery, ischemia will occur and orchiectomy is then needed.

A 15-year-old female dancer presents with 4 weeks of right forefoot pain. Physical exam reveals tenderness over the second metatarsal head. Xray shows sclerosis and flattening of the second metatarsal head. The most likely diagnosis is ? (A) Morton's Neuroma (B) Freiberg Disease (C) Kienbock Disease (D) Kohler Disease

B - The correct answer is B. Freiberg disease is a painful condition of the forefoot occurring most often in adolescent girls who participate in ballet and dance. The exact etiology is unknown, but it is commonly characterized by disordered ossification of the second metatarsal head. Possible causes include trauma, repetitive stress, disruption in blood supply, or improper shoe wear. Bilateral involvement occurs in less than 10 percent of patients.Pain is exacerbated by weight-bearing and athletic activities. Point tenderness and swelling are commonly found over the affected metatarsal head. Plain radiographs show sclerosis and varying degrees of flattening of the affected articular surface. In most patients, nonoperative treatment consisting of activity modification, metatarsal pads, and well-padded shoes facilitates regeneration of the metatarsal head and spontaneous resolution of symptom. Köhler disease is an osteochondrosis of the navicular bone in the foot. Patients often present between two and eight years of age, and boys are three to five times more likely to be affected. Morton's Neuroma is a perineural fibrosis and nerve degeneration of the common digital nerve. It is not a true neuroma, although it results in neuropathic pain in the distribution of the interdigital nerve secondary to repetitive irritation of the nerve. The most frequent location is between the third and fourth metatarsals (third webspace). Other, less common locations are between the second and third metatarsals (second webspace) and, rarely, between the first and second (first webspace) or fourth and fifth (fourth webspace) metatarsals. There are no xray findings indicative of Morton's Neuroma. Kienböck disease is a condition of uncertain etiology that results in osteonecrosis of the carpal lunate.

You are the team physician for a college football team. Per your usual routine, you are attending the first practice during fall camp when the team has put on full pads. It is a warm and humid August day with a heat index in the caution zone". Toward the end of practice, ons of the linebackers wanders over towards the the sideline and collapses to the ground. The athletic trainers quickly ask you to come over for an evaluation. You have an AED present but no other diagnostic equipment on the sidelines. Your evaluation shows the athlete to have a decreased level of consciousness but able to be aroused by loud voice and moderate shaking. He is very confused and cannot tell you where he is or the correct date. His skin is hot and covered with sweat and his uniform is soaked. You can feel a femoral pulse that is weak and rapid but regular with a rate of about 140. His respirations are 20. Which of the following is your best next order to the athletic training staff? (A) Hook up the AED and call 911 for transport to the closest ED (B) Place the athlete in tub of ice water and call 911 for transport to the closest ED (C) Send one of the trainers back to the training room (it's only 5 minutes round trip to get a thermonmeter and BP cuff. Move the athlete to the shade until the trainer gets back (D) Send one of the trainers back to the training room for IV supplies. Move the athlete to the shade and start an IV with NS at a wide-open rate as soon as the traininer gets back. Call 911 if the athlete doesn't respond within 20 minutes to this initial resuscitation (E) Immobilize his C-spine and perform a SCAT 3

B - The correct answer is B. The first day in full pads during fall camp is a high-risk time for heat injuries so the index of suspicion must be very high. The fact that the athlete is clearly demonstrating a decreased level of consciousness and is very confused suggests that he is suffering from heat stroke. The best treatment for heat stroke is early cooling with an active, whole-body cooling method. Choice (A) is incorrect because it is less likely that he is having an abnormal heart rhythm that the AED will choose to defibrillate. Choice (C) is not as good a choice as (B) because it delays active cooling. It would be acceptable to send a trainer for extra equipment while you are cooling the athlete in a tub of ice water. Choice (D) also delays active cooling and it is unlikely that hypotension is the primary cause for the athelte's collapse due to the palpable femoral pulse. Cinally choise (E) is reasonable answer since collapse and confusion can be a symptom of concussion as well. Nevertheless, on a hot day, heat injury should be the first thought unless the change in mental status occurs immediately after a witnessed collision.

Fractures in the long bone physes of the extremities may result in interruption of growth in the pediatric population. Which long bone physis is responsible for the most longitudinal growth in the lower extremityper year? (A) Proximal femur (B) Distal femur (C) Proximal Tibia (D) Distal Tibia (E) Proximal Fibula

B - The proximal femur provides approximately 3.5 mm/yr of growth, while the distal femur provides 9 mm/yr. The proximal tibia's average growth is 6 mm/yr and the distal tibia's is 5 mm.yr. This is information to know when treating physeal fractures in the lower extremity.

You are asked by your high school trainer to see a 9th grade football player after he collapsed at the end of practice over the weekend. You examine him in the high school training room on Monday because he "can't get a ride to the office." He states that this has never happened to him before, it was very hot that day and he hadn't eaten anything all day before practice. He denies any funny heart beats and doesn't know his family history. He feels completely normal now. You review his PPE done by an outside facility which states that he had an uncle that drowned at 27 on his mother's side, he has been raised by his mother so there is no family history from his father's side and his cardiac exam was unremarkable. On your physical exam he has a II/VI holosystolic murmur heard best at the left sternal boarder. The murmur improves when he is supine and gets worse when you ask him to "bear down." You call his mother to inquire about the uncle who drowned and about the father's family history. She is unavailable and the athlete is anxious to get to practice so he doesn't miss anything. You next step is? (A) Allow him to go to practice, but make him noncontact until he is able to get an EKG (B) Fully restrict him, order an EKG, and pursue his family history (C) Clear him to return to play, but order and echocardiogram (D) Clear him to return to play, as this was a response to the heat and lack of food, but make it very clear to him that he has to eat before practice and take water breaks when he gets thirsty

B - This case is concerning for Hypertrophic Cardiomyopathy (HCM). There is exertional collapse, a + family history of unexplained drowning, and a murmur that is consistent with HCM (although the majority of athletes with HCM will not have a classic murmur). He should be fully restricted until his evaluation is complete (Answers A, C and D are incorrect). He may require an echocardiogram, Holter, or other investigations in the future, but the first step is to get an EKG, follow up on his family history, and base future testing off of those results.

A 20-year-male presents to clinic complaining of mild diffuse pain in his knee and difficulty climibing stairs for the last week. The pain first started after he was tackled while playing recreational tackle football with some friends. He is unsure exactly how he landed, but thinks that he may have landed directly on a flexed knee. Physical exam reveals trace effusion, a positive posterior sag, as well as a positive posterior drawer test. What other test is essential to do in order to determine the stability of this injury? (A) Thomas test (B) Dial test (C) McMurray test (D) Thompson test (E) Squeeze test

B - This patient has a posterior cruciate ligament injury by history and physical exam, however it is also essential to determine whether or not this is associated with a posterolateral corner injury because early sugical repair is indicated for Grade II and Grade III injuries. The posterolateral corner consists of the lateral collateral ligament, the popliteus tendon, and the arcuate ligament complex. A. is incorrect because the Thomas test evaluates for a flexion deformity of the iliopsoas, C. is incorrect because this test evaluates for meniscus tear and although a meniscal tear may also be present, it does not determine the stability of the knee, D. is incorrect because the Thompson test is used to evaluate the integrity of the Achilles tendon, and E. is incorrect because the squeeze test evaluates for syndesmotic injuries of the ankle. B. is the correct answer. The dial test involves placing the patient in the supine or prone position and passively rotating the tibia on the femur at 30 and 90 degrees of flexion and comparing to the uninjured side. If there is increased rotation on the injured side at 30 degrees, injury to the posterolateral corner is likely.

You examine a boxer at the ringside who is complaining of blurred vision in his right eye after being hit with a left hook. With a penlight, you are able to visualize blood on the dependent area of his right eye's anterior chamber. With a slit lamp, you are able to confirm this location, and blood over the inferior angle. Which of the following is true of this injury? (A) Corneal blood staining usually occurs a few hours after the initial bleed, and then clears within a few days (B) Intraocular pressure may increase due to obstruction of the trabecular meshwork by red blood cells, fibrin, and cellular debris (C) Medical management includes mandatory rest, an eye patch, and oral steroids only (D) Rebleeding may occur, but usually more than 7 days after the first bleed

B - When intraocular pressure is high and corneal endothelial dysfunction ensues, corneal blood staining can also occur, and this usually takes several months to years to clear, sometimes requiring a corneal transplant [1]. Acute elevation of the intraocular pressure is the most common consequence of a hyphema [1,2]. The elevation is directly proportional to the amount of red blood cells, fibrin, and cellular debris after the injury [1]. Medical management consists of mandatory rest, an eye shield, aqueous suppressants, and oral and topical steroids [2]. In 38% of patients, rebleeding occurs, usually within the first 7 days of the initial bleed [2].

During a pre-participation exam on a grade school student athlete, which of the following information is needed to appropriately interpret the athletes blood pressure? (A) Normative data of blood pressure readings, the childs age, weight, and height (B) Normative data of blood pressure readings, the childs age, gender, and height (C) Normative data of blood pressure readings, the childs age, weight, and gender (D) Normative data of blood pressure readings, the childs weight, height and gender

B is the correct answer: The definition of hypertension in children and adolescents is based on the normative distribution of BP in healthy children. Normal BP is defined as SBP and DBP that are <90th percentile for gender, age, and height. Hypertension is defined as average SBP and DBP that is >95th percentile for gender, age, and height on at least 3 separate occasions. A is incorrect because it does not include a childs gender C is incorrect because it does not include a childs height D is incorrect because ti does not include a childs age The definition of childhood hypertension (HTN) is based upon the normative distribution of blood pressure (BP) in healthy children. This is in contrast to adult HTN, which is primarily defined by clinical outcome data. Body size is the most important determinant of BP in children and adolescents. Thus, classification of BP is more accurate when the values are adjusted for height as well as age and gender to avoid misclassifying children at the extremes of normal growth.

A 23-year-old college football player presents to you on the sideline during a game with a complaint of right knee pain. He went straight down without being hit due to immediate pain. He complains of diffuse pain rated 7 out of 10 and a sensation of instability. On exam you note knee effusion, limited range of motion, non-tender tibia, a positive ballottement test, and guarding during Lachman test and pivot shift test. What is the most likely diagnosis based on this information? (A) Medial collateral ligament injury (B) Posterior cruciate ligament injury (C) Anterior cruciate ligament injury (D) Tibial plateau fracture

C - A plant-and-cut maneuver is a common mechanism of injury for an anterior cruciate ligament rupture, and typically results in immediate pain, subjective instability and hemarthrosis (swelling and positive Ballottement test). Anterior cruciate ligament injuries are found in 62%-77% of acute knee hemarthoses; although Lachman's test was negative, it is often difficult to determine joint stability in the acute phase. Although tibial plateau fractures may also result in hemarthosis after twisting injuries, the most common mechanism of injury is a direct axial load. They are a much less common cause of acute knee hemarthoses. A medial collateral ligament injury can result from valgus strain during a cut or a medially-directed external blow; however, similar to tibial plateau fractures, it is a much less common cause of acute hemarthosis. The pain would be expected to be more focally located at the medial aspect of the knee. A posterior cruciate ligament injury is a much less common cause and the mechanism would typically involve knee hyperflexion or a posteriorly-directed external blow to the tibia.

A 16-year-old female high school soccer player complains of shortness of breath during exercise that resolves within a few minutes of stopping activity. Using albuterol when her symptoms occur is not helpful. While examining her in the training room she is calm and breath sounds are equal and clear. The following day during practice she is noted to have loud breathing during inspiration and a hoarse voice. Which of the following diagnostic test results are most likely to be present in this patient? (A) Esophageal pH probe evidence of laryngeal reflux (B) Decreased FEV1 on resting spirometry (C) Laryngoscopy with visualization of vocal cord adduction on inspiration (D) Positive methacholine challenge

C - Answer 3 is the correct answer. Vocal cord dysfunction(VCD)is more common in female than male athletes. Lack of clinical improvement with short-acting bronchodilator makes exercise induced bronchospasm (EIB) less likely. The history of dyspnea that improves shortly after exercise ceases is consistent with VCD, whereas EIB may worsen in the few minutes after the patient stops exercising. Stridor and voice hoarseness further support the diagnosis of VCD. Laryngoscopy is the gold standard for diagnosis of VCD with visualization of paradoxical adduction of the vocal cords during inspiration. Decreased FEV1, answer 2, on resting spirometry is present in chronic asthma. A positive methacholine challenge, answer 4, can be present with asthma or EIB, but is negative in VCD. Finally esophageal pH probe, answer 1, is used for the diagnosis of GERD which may be a contributing factor in VCD, but is not necessary for the diagnosis.

Which of the following statements are true concerning an athlete with suspected female athlete triad syndrome? (A) The syndrome is currently defined as disordered eating, amenorrhea, and osteoporosis (B) The treatment goal of exogenous estrogen (OCPs) is to re-establish menses in amenorrheic females (C) Cognitive behavioral therapy has been shown to be the most efficacious therapy for eating disorders (D) T-scores should be used to classify the patient with osteopenia or osteoporosis (E) A greater than 50% reduction in exercise load is needed to reverse reproductive dsyfunction

C - Answer A was the old definition of the syndrome. The current components of the female athlete triad syndrome are: energy availability, menstrual dysfunction, and bone health. Each component has a spectrum of pathology that includes the old components but is not limited to them. Answer B is incorrect as OCPs are given to raise estrogen levels to prevent further bone loss. Answer C is the correct choice. The treatment of the syndrome is multifactorial. Answer D is incorrect as Z-scores should be used in young patients to classify bone mineral density as low for chronological age or below expected range for age not osteopenia or osteoporosis. Answer E is incorrect as only a 10% reduction in exercise load in either intensity or duration is sufficient to reverse reproductive dysfunction.

A 20-year-old college football player is blindsided and goes down on the field. He is complaining of severe right sided chest pain and shortness of breath. Upon removal of his helmet and pads, you notice that he is diaphoretic, he has increased jugular venous distention (JVD), and his trachea appears to be shifted to the left of the midline. Which of the following would be most appropriate at this time? (A) Allow the player to return to play (B) Provide oxygen and allow the player to rest on the sideline for the remainder of the series of plays (C) Perform immediate needle decompression with a large needle inserted in the anterior chest wall into the second intercostal space at the midclavicular line (D) Provide oxygen, monitor blood pressure and heart rate, and have the player transported to a medical facility

C - C is the correct answer. A tension pneumothorax is a life-threatening condition that develops when air is trapped in the pleural cavity under positive pressure, displacing mediastinal structures and compromising cardiopulmonary function. This can be secondary to blunt trauma to the chest wall as in this case. Prompt recognition of this condition is life saving. Because tension pneumothorax occurs infrequently and has a potentially devastating outcome, a high index of suspicion and knowledge of basic emergency thoracic decompression procedures are important for all healthcare personnel. Immediate decompression of the thorax is mandatory when tension pneumothorax is suspected. This should not be delayed for radiographic confirmation. This is performed by inserting a large needle in the anterior chest wall into the second intercostal space at the midclavicular line. Allowing the player to return to play and providing oxygen and allowing the player to rest on the sideline for the remainder of the series of plays, therefore, are incorrect (A and B). Providing oxygen, monitoring blood pressure and heart rate, and having the player transported to a medical facility would be appropriate in the setting of a non life threatening pneumothorax, not in the setting of a tension pneumothorax.

Differentiating medial epicondylitis(ME) from other sources of elbow pain can be challenging. Which of the following maneuvers is suggestive of ME if it provokes symptoms? (A) Shoulder abducted to 90, and elbow flexed at 90, with active supination against resistance in thumbs up position (B) Resisted wrist extension with the elbow extended (C) Resisted wrist flexion, radial deviation, and forearm pronation (D) Wrist flexion while opposing the thumb and small finger

C - C is the correct answer. ME tendonosis can be identified by pain elicicted just distal to the medial epicondyle at the pronator teres and flexor carpi radialis with resisted wrist flexion, radial deviation, and forearm pronation. A is incorrect as this is a description of the "Hook Test" for distal bicep rupture B is incorrect as this is the description for identifying lateral epicondylitis by stressing the wrist extensors to provoke pain in the lateral epicondyle. D is incorrect as this is the Schaeffer test for identifying the palmaris longus tendon, frequently used in reconstruction of an MCL but absent in over 10% of the population.

A collegiate diver entered the pool awkwardly on a dive and felt a pop in her ear. This was followed by pain and associated with nausea and dizziness. You diagnose her with a ruptured tympanic membrane. Which of the following is true regarding this injury? (A) Appropriate headgear and diving techniques have no role in prevention of this injury (B) Athletes participating in land-based sports have no need for topical antimicrobial therapy (C) Use of custom- made "molded" earplugs assist in keeping ear canal dry upon return to the water (D) Surgical repair of the tympanic membrane is typically necessary to allow a return to sports

C - C is the correct answer. Molded earplugs are useful in keeping the ear canal clean and dry when athletes return to the water after tympanic membrane rupture. These injuries are at high risk for infection so antimicrobial treatment is necessary, regardless of the sport (B). Most of the injuries heal with conservative measures (D) and appropriate equipment and technique may prevent these injuries (A).

A 30-year-old Martials Arts champion comes to you after a tournament complaining of hypothenar pain and numbness in his dominant hand. On physical exam you find hypothenar tenderness to palpation with blanching and coolness of the third, fourth, and fifth digits over his dominant hand. Which of the following tests would most likely help confirm the diagnosis? (A) Nail Fold Capillaroscopy (B) Cold Stimulation Test (C) Allen Test (D) Elevated Arm Test

C - C is the correct answer. This patient most likely has Ulnar or Hypothenar Hammer Syndrome, first described in a factory worker by von Rosen in 1934. This is characterized by post-traumatic digital ischemia caused by distal ulnar artery occlusion. It is more common in male patients, usually in the dominant hand, and it arises from using the hypothenar part of the hand as a hammer. Patients often present with hypothenar pain with cool, pale 3rd, 4th and 5th digits. Absence of the triphasic color change tenderness and an abnormal Allen test (sluggish or complete absence of filling of the hand with the radial artery occluded) help confirm the diagnosis. Doppler and arteriographic studies are required for confirmation. A is incorrect. One diagnostic test useful in helping to determine the correct form of For will be abnormal for those who have the secondary form. B is incorrect. A cold stimulation test can be used to trigger Raynaud's symptoms. For this test, a small device that measures temperature is taped to cold, and the device measures how quickly the fingers return to their normal temperature. If the patient has Raynaud's, it may take more than 20 minutes for the fingers to return to their normal temperature. D is incorrect. Many examination maneuvers have been described for diagnosing Thoracic Outlet Syndrome. One of the most discussed tests, the elevated arm stress test, is performed by having the patient abduct and externally rotate the arms, both to 90 degrees, and followed by opening and closing sthesia, and possibly other vascular symptoms.

A 20-year-old right-handed pitcher presents to your office for chronic right posterior elbow pain with throwing. You suspect valgus extension overload syndrome. Which of the following is most characteristic of this condition? (A) Posteromedial elbow pain during the acceleration phase of throwing (B) Posterolateral elbow pain during the acceleration phase of throwing (C) Posteromedial elbow pain during the deceleration phase of throwing (D) Posterolateral elbow pain during the deceleration phase of throwing

C - C is the correct answer. Valgus extension overload (VEO) is a syndrome of symptoms and exam findings seen commonly in the throwing athlete. It has been shown that excessive valgus stress and medial elbow laxity results in posteromedial impingement and osteophyte formation at the posteromedial olecranon over time. Therefore, pain typically occurs in the posteromedial elbow during terminal extension in the deceleration and follow-through phases of the throwing motion (C). Medial elbow pain during the acceleration phase is more likely associated with ulnar collateral ligament problems (A is incorrect). Posterolateral pain can result from lateral compressive forces and may be related to stress fracture of the olecranon, but this is not characteristic of VEO (B and D are incorrect).

A 20-year-old college junior swimmer presents to the training room complaining of painful urination and purulent discharge from his penis. He admits to having multiple female sexual partners without the use of a condom. You order a nucleic acid amplification test of his urine which reveals infection with both N. gonorrhea and C. trachomatis. Which of the following statements about sexually transmitted diseases is correct? (A) Treatment with a single dose of ceftriaxone (Rocephin) 125 mg IM is sufficient to treat both infections (B) Treatment with a single dose of benzathine penicillin G (Bicillin L-A) 2.4 million units IM should be given empirically to treat primary syphilis (C) The sexual partners of the patient should be notified of a possible exposure to a sexually transmitted disease, and the patient should be offered testing for other possible infections (D) The patient should be excluded from athletic competition while undergoing treatment because of therisk of spread via contact from one athlete to another. (E) A "test-of -cure" is not required as drug resistance has not been seen in treatment of gonorrhea

C - Ceftriaxone continues to be an effective treatment of N. gonorrhea urethritis; however, evidence of bacterial resistance is emerging with a few documented cases of true resistance. For this reason a "test of cure" is recommended and if testing returns positive after treatment, cultures should be sent to evaluate for resistance. Ceftriaxone is not the preferred treatment of C. trachomatis (1 g single dose of azithromycin, observed) and should not be relied upon to cure chlamydial infections. Co-infection rates can be as high as 46% with gonorrhea and chlamydia, so double treatment should be considered if one is diagnosed and the other was not tested. Benzathine penicillin G 2.4 million units for one dose IM is the treatment of choice for primary syphilis, however, there is no current evidence this patient is infected with T. pallidum. There is no reason to exclude the athlete from competition for bacterial urethritis. All sexual partners of the athlete should be notified, and the athlete should be offered testing for other sexually transmitted infections such as HIV and syphilis, as the risk of contracting these infections is higher in those diagnosed with other sexually transmitted diseases.

breath and mouth swelling stating she was stung by a bee a few minutes earlier. You quickly examine her and notice angioedema to her tongue and diffuse wheezes, tachypnea, and the ability to speak in onetwo word phrases only. Which of the following is the most appropriate next step? (A) Provide intramuscular diphenhydramine 50mg, 100mg of ranitidine, and 60mg of prednisone orally and observe for two hours in the training room (B) remove the patient's clothing and immediately decontaminate her in the shower (C) inject 0.3 mg of epinephrine intramuscularly and call 911 (D) provide 50mg of oral benadryl, 60mg of oral prednisone and call 911 (E) Take a detailed history regarding prior episodes, medications, allergies, family history, and new exposures (medications, foods, clothing)

C - Correct answer: (C) This patient is clearly experiencing anaphylaxis, a potentially fatal condition in minutes if not immediately treated with the medication of choice: epinephrine. Glucocorticoids, antihistamines, and bronchodilators are adjunctive therapies, but are not substitutes for epinephrine and should not be administered alone. Thus, options (A) and (D) are incorrect. Avoiding the potential trigger is important for prevention, but taking the time to decontaminate delays life saving treatment with epinephrine and is not recommended, making option (B) incorrect. In this setting, taking the time to take a detailed history delays treatment as well and is not the most appropriate initial action, making option (E) incorrect.

Which of the following methods has been shown to be the most effective way of cooling most athletes with serious heat illness? (A) Packing in ice (B) Placing ice bags in the axilla and groin (C) Ice water immersion (D) Ice water ingestion

C - Damage related to time core temperature exceeds 40°C for more than 30 minutes. Thus speed of cooling a person down with severe heat illness is critical. Ice water immersion has found to be the most effective method of cooling. However, cold water immersion is nearly as effective and is a reasonable alternative. In general, the other answers do decrease body core temperature, albeit at a significantly lower rate and should only be used when no other options are available.

A snowboarder presents with right shoulder pain after sustaining a direct blow to the acromion with the humerus in an adducted position. Which of the following best supports non-surgical management of this case? (A) Absence of a fall on an outstretched arm or elbow (FOOSH) decreases the likelihood of severe injury (B) Weighted radiographs are required to make a determination of surgical versus non-surgical management of this case (C) Tenderness over the AC joint, moderate distal clavicular prominence and pain at the distal clavicle with slight widening of the AC joint on radiograph (D) An obvious prominence of the distal clavicle is suggestive of Type III injury

C - The answer is C. The classic cause of an AC joint injury is a direct blow to the acromion with the humerus in an adducted position, but may be caused by indirect trauma, such as falling on an outstretched arm or elbow. Weighted radiographs have been a part of the traditional work-up, but their need has come into question. With type II injuries, the distal clavicle may be more prominent on inspection and the patient may have pain at the distal end of the clavicle from the sprained coracoclavicular ligament. Radiographs may be normal or may show slight widening of the AC joint. Management of this injury is non-operative. Type III injuries usually present with obvious visible prominence of the distal clavicle. There is consensus that type I and II injuries are treated nonoperatively, while types IV, V, and VI are treated surgically. The treatment of type III injuries remains an area of controversy.

A 16-year-old high school football player presents to clinic complaining of two days of sore throat and malaise. Physical exam is remarkable for fever to 102F, tachycardia to 105, tonsillar hypertrophy, and posterior lymphadenopathy. Laboratory tests will not be available for several days, but he is eager to return to play. Based on your presumptive diagnosis, which of the following is true? (A) The patient may safely return to play in 2 weeks (B) The spleen is enlarged in approximately 10% of cases (C) Splenic rupture may occur in 0.1-0.2% of all cases (D) Splenic size reaches its peak approximately 6 weeks after diagnosis (E) Physical exam of the abdomen is highly sensitive to screen for patients who are at risk for rupture

C - Infectious mononucleosis is a common condition with an incidence of 1-3% in 15-21 year olds. A high percentage close to 100% according to some sources will have splenomegaly with this disease. While the absolute enlargement and rate of growth is variable, the spleen peaks in size approximately 12 days after diagnosis. Regress is more predictable at 1% per day after peak size. Physical exam of the abdomen features poor sensitivity (20-70%) but better specificity (70-100%) for splenomegaly. The decision to allow return to play is influenced by spleen size, but 2 weeks (around the time of the peak) would be considered too short by most clinicians. The current consensus from the literature suggests return to light contact at minimum 3 weeks after diagnosis. The majority of splenic ruptures happen within the first three weeks, though some have been reported up to 7 weeks out. The rate of rupture is approximately 0.1-0.2% of all cases.

A 10-year-old female gymnast presents to the office with an intermittent history of left knee pain and swelling for the last 3-4 months. She has no known trauma or recent injury. Hard landings tend to bother her the most. She does get some relief from icing and taking ibuprofen. Her coaches and parents are concerned because her knee pain is worsening instead of improving and she has a big regional meet coming up in a few weeks. Ideally she wants her knee pain gone before the meet. Physical exam only reveals tenderness to the distal portion of the medial femoral condyle. Radiographs of the left knee are show Osteochondritis Dessicans. Which of the following is the best treatment option for this young athlete? (A) Relative rest, physical therapy, ice and anti-inflammatories as needed. Participation in continued activities as tolerated (B) Complete rest for 2 weeks and then return to activities as tolerated (C) Complete rest and immobilization with a hinged knee brace locked in extension for 6 weeks. An MRI should also be obtained to stage the lesion (D) Referral to a pediatric orthopedic surgery for drilling and fixation of the lesion

C - Osteochondritis dessicans is characterized by a focal area of subchondral bone that undergoes necrosis. The overlying cartilage remains intact to variable degrees, receiving nourishment from the synovial fluid. As the necrotic bone is resorbed, the cartilage loses its supporting structure. Subsequently, the bony fragment may be displaced into the joint space. It is the most common cause of a loose body in the joint space in adolescents and may lead to considerable debility. Historically, it has been seen predominantly in young men. Recently, it has been reported with increasing frequency among young female athletes. Clinical findings may be subtle including vague knee pain, intermittent swelling, and loss of motion. OCD is a radiologic diagnosis and anteroposterior, lateral and tunnel-view (knee in flexion) radiographs should be obtained. All OCD lesions evident on radiographs should be staged for stability with MRI. Conservative treatment of stable lesions is generally accepted in girls younger than 11 years old and boys younger than 13 years old. Treatment consists of limitation of activity and immobilization. Recommend a trial of nonoperative treatment for 3-6 months. If symptoms persist or failure to unite is observed, proceed with surgical treatment. Prognosis worsens with age and physis closure. Therefore, the goal of management of juvenile OCD is to promote resolution of the lesion before physis closure. Unstable lesions at any age must be referred for surgical management. A is incorrect because this treatment could lead to worsening symptoms and the defect becoming larger. B is incorrect because the rest period needs to be significantly longer. D is incorrect because younger children have an excellent prognosis when treated conservatively and will rarely need surgery.

A 9-year-old female level 7 gymnast presents to the pediatric orthopaedic clinic with a 1 month history or left lateral sided elbow pain. The pain acutely worsened 2 days prior to presentation when she pushed off with that arm and felt a pop while doing a round-off back handspring. She has noticed some mild swelling and has decreased range of motion on exam today. Results of radiographic evaluation show osteochondritis dissecans of the elbow. How do you counsel this athlete and her mother based on current literature? (A) Recommend 3- 4 months of rest with no weightbearing or strenuous activity on the left upper extremity. If pain free at that time, gradually progress back to previous level of activity (B) Recommend rest until pain resolves and then begin a gradual progression back into activity as long as she is able to tolerate this pain free (C) Recommend obtaining an MRI or MR Arthrogram to evaluate the stability of the lesion and if unstable refer to sports orthopaedist for microfracture or other restorative procedure (D) Recommend retiring from gymnastics completely as this lesion is an indication that she is not able to tolerate the demands that gymnastics is placing on her joints. If she continues with gymnastics this lesion will not heal with or without surgical intervention and lead to chronic pain and limited function

C - Osteochondritis dissecans of the elbow is one of the causes of elbow pain in overhead athletes. It typically affects children between 8 and 14 years of age. The most common site in the elbow is the capitellum, but it can also involve the radial head and trochlear groove. Most commonly patients will present with lateral sided elbow pain that is progressive with activity and often associated with swelling particularly after activity. Radiographic evaluation is generally confirmatory although in the early stages, plain radiographs may be normal. X-rays will demonstrate a darkened crater shaped area on the weight bearing portion of the capitellum. Radiographs are more commonly followed by an MRI or MRA to evaluate for stability of the lesion as multiple recent studies have shown instability at initial presentation to be a poor predictor of healing with conservative management (Answer C). An osteochondritis dissecans lesion with an open capitellar physis and good range of elbow motion also result in good outcomes with conservative management. In patients with a closed capitellar physis, surgery provided significantly better results than elbow rest. Sport activity has also been shown to effect outcomes with gymnasts having poorer outcomes with conservative management in comparison to other upper extremity athletes. There is fairly good consensus in conservative treatment with a period of rest for 3-4 months with reevaluation at that time for athletes with stable lesions who have an open physis. Therefore A is incorrect because we do not yet know if the lesion is stable. B is incorrect because she should rest for at least 3-4 months if the lesion is stable. D is incorrect because with proper management she will most likely make a successful return to gymnastics.

The day after you fly to Colorado to hike high in the Rocky Mountains you develop flu like symptoms including headache, nausea, fatigue, and difficulty sleeping. Being the astute fellow you are you recognize the symptoms of acute mountain sickness. Unfortunately only then do you remember that the following is the most important measure to prevent the onset of altitude illness? (A) Acetazolamide (B) Dexamethasone (C) slow ascent (D) Theophylline

C - Symptoms of acute mountain sickness and early high-altitude cerebral edema include headache and at least one of the following: anorexia, nausea or vomiting, dizziness or lightheadedness, difficulty sleeping, and fatigue or weakness. These symptoms can easily be misinterpreted as a viral illness. The key criteria, however, are a recent gain in altitude and (although not specifically stated) the absence of other causes of the symptoms. Symptoms of acute mountain sickness typically occur within six to 12 hours of gaining altitude, and range from mild with spontaneous resolution (especially at altitudes less than 11,400 ft [3,500 m]) to severe with progression to high-altitude cerebral edema. The most accepted method of preventing acute mountain sickness and high-altitude cerebral edema is to ascend slowly. However, many climbers have difficulty following this advice. Acetazolamide, a carbonic anhydrase inhibitor, may be used as prophylaxis ; it should be started at least one day before climbing and continued until acclimatization at the highest sleeping elevation. Dexamethasone is also effective for prophylaxis and treatment of acute mountain sickness as a second-line agent, but it does not assist in acclimatization and therefore may lead to rebound acute mountain sickness when it is discontinued. A small study showed that low-dose theophylline is also beneficial for prevention of acute mountain sickness.

A patient with a diagnosis of lateral epicondylitis fails to respond to conservative management and is referred to your office for evaluation. By history, she has vague deep forearm pain and on exam, she has weakness with extension of the wrist, thumb but most notably the fingers. She is able to make an "O" with her thumb and forefinger and has normal wrist flexion. Sensation is intact over the lateral elbow. Entrapment of which nerve is most likely? (A) Anterior Interosseus Nerve (B) Ulnar Nerve (C) Posterior Interosseus Nerve (D) Musculocutaneous Nerve

C - The answer is 3. The posterior interosseus nerve, a branch of the radial nerve, innervates the following extensor muscles: extensor carpi radialis brevis, extensor digitorum, extensor digiti minimi, extensor carpi ulnaris, supinator muscle, abductor pollicis longus, extensor pollicis brevis, extensor pollicis longus and extensor indicis. Posterior interosseous neuropathy is purely a motor syndrome resulting in finger drop and radial wrist deviation on extension. #1 is incorrect because the anterior interosseus nerve, a branch of the median nerve, innervates the flexor pollicis longus, deep flexors of digits 2 and 3, and the pronator quadratus. The patient would not be able to make the "O" with anterior interosseus nerve dysfunction. #2 is incorrect because the ulnar nerve innervates the following muscles in the forearm and hand: flexor digitorum profundus (medial half), opponens digiti minimi, abductor digiti minimi, flexor digiti minimi, the third and fourth lumbrical muscles, dorsal interossei, palmar interossei, adductor pollicis, the flexor pollicis brevis (deep head) and the palmaris brevis. With ulnar nerve entrapment, wrist or finger extension would not be affected. #4 is incorrect because the musculocutaneous nerve, comprised of fibers from C5, C6, and C7, innervates the coracobrachialis, biceps and brachialis muscle. It also supplies sensation to the lateral forearm. Injury to this nerve would result in weakness with flexion of the elbow and decreased sensation over the lateral elbow.

A 25-year-old male quad rugby player with C5 tetraplegia complains of headache and facial flushing acutely during a local game. His injury was six months ago and he is still adjusting to life with a spinal cord injury. He denies knowledge of a specific injury during this game, and is unsure of what is causing his symptoms. On exam, you find that his blood pressure is 180/100, heart rate 70. His face and neck are flushed and sweating, but hands are not sweating. You also notice nasal congestion, which was not present prior to the game. You diagnose the following: (A) Essential hypertension (B) Hypertensive urgency (C) Autonomic dysreflexia (D) Boosting

C - The correct answer is C. Autonomic dysreflexia (AD) can present for the first time up to a year after injury, though patients will often experience their first episode while still hospitalized. AD can be seen in patients with spinal cord injuries (SCI) at the T6 level or above. Signs and symptoms include hypertension, relative bradycardia, headache, nasal congestion as well as diaphoresis, flushing and piloerection above the level of the lesion. Noxious stimulation below the level of the lesion results in an exaggerated sympathetic response and release of catecholamines causing vasoconstriction and hypertension. The brain is not able to provide central inhibitory control of this sympathetic reflex in the presence of a high level spinal cord injury. Parasympathetic response above the level of the lesion leads to bradycardia and vasodilation, but it is usually not sufficient to lower the BP. Autonomic dysreflexia represents a medical emergency. Common causes include bladder or bowel distension, tight fitting garments or excessive pressure below the level of the lesion. Abdominal or urological issues (cholecystitis, appendicitis) and MSK issues can also precipitate AD. The goal is prompt removal of the offending stimulus if able, or treatment of the blood pressure until the cause is discovered and treated. . In this athlete, first line should be to check his bladder catheter, be sure no clothing is too tight, and be sure he has not suffered an injury during the game to a non-sensate area before looking for other causes. A and B are incorrect. He has a relative bradycardia in the face of hypertension, and the distribution of symptoms above the level of his spinal cord injury point to AD as the cause. Treatment only of the hypertension, without identifying the underlying cause will not be sufficient. D is incorrect. Boosting is a method whereby competitive athletes with SCI will intentionally cause AD for an advantage in sports. There is no evidence that this athlete has intentionally caused his symptoms. The most common method of boosting is through bladder distension by clamping the bladder catheter and ingesting large amounts of fluids. This does provide an unfair advantage to the athlete, and is also a dangerous practice for the athlete, and is therefore banned by the International Paralympic Committee.

In patients presenting to you with scoliosis, which of the following should not highten your concern forprogression? (A) Female gender (B) Pre-pubertal age (C) Single curves (D) Risser Index less than two

C - The correct answer is C. Single curves carry less risk of progression than double curves. The highest risk of progression is in the adolescent years, and especially during the growth spurt (Answer B). Risser indexes are based on a 1-5 scale and reflect the level of ossification and overall maturity at the iliac crest laterally to medially. Risser indexes of 1 are the least mature and suggest the highest risk for progression of scoliosis, while 5 represent skeletal maturity and the least risk (Answer D). Females progress more often than males (Answer A).

Which of the following is correct regarding the patellar fat pad? (A) The infrapatellar fat pad is located distal to the patellar tendon (B) Fat pad irritation is exacerbated by flexion of the knee (C) Fad pad impingement is painful because it is a highly innervated structure (D) Surgical excision is often necessary for definitive treatment of an irritated fat pad

C - The infrapatellar fat pad is a highly innervated structure located at the inferior pole of the patella, posterior to the patellar tendon. Irritation or impingement can be caused by either a direct blow or due to hyperextension of the knee. People with fat pad irritation have exacerbation of the pain with extension of the leg (straight leg raises, prolonged standing. Treatment is often taping the knee either at the superior aspect of the patella to lever the inferior pole anteriorly or just distal to the fad pad to help support it.

A 20 year old hockey player presents to the athletic training room with the rash demonstrated in the image. He reports that the rash has varied in severity, is located under the shoulder pads and has failed to respond to local cleansing. It is mildly pruritic. Which is the likely diagnosis? (A) Herpes Gladiatorum (B) Tinea Corporus (C) Contact Dermatitis (D) Molluscum Contagiosum

C - The most correct answer is Contact Dermatitis. Rashes are less common in hockey than many providers experience in the combative arts as in wrestling, judo and mixed martial arts. The demonstrated rash does not have the discrete dome-shaped and umbilicated lesions commonly seen in Molluscum Contagiosum or the oval erythematous, raised lesions w mild central scale and intradermal edema of the periphery as seen in tinea corporus. Although herpes Gladiatorum can appear like this lesion, given the pruritis and the location under the pads in a lower risk sport makes it much less likely.

A 22-year-old collegiate male rower presents with a six week history of progressive left lateral chest wall pain. He denies any antecedent trauma, and describes the onset of the pain as gradual. Initially the pain was intermittent and triggered by rowing, but during the past two weeks the pain has become constant and is exacerbated by coughing and rolling over in bed. The pain has prevented the athlete from rowing for the last 3 days. He denies fevers, night sweats, weight loss, dyspnea, and hemoptysis. On physical exam, the athlete has normal vital signs. He is well nourished and well hydrated but appears uncomfortable. Palpation yields focal tenderness over the posterolateral angle of the left 6th rib. A squeeze test is positive. The remainder of the physical examination, including auscultation of the chest, is normal. Standard plain films of the chest are obtained and are read as negative. In order to provide the athlete with a definitive diagnosis and accurately provide recommendations on return to play, you decide to order further testing. Which of the following would be the most helpful next diagnostic test? (A) Computed tomography (CT) of the chest (B) Serum lactate dehydrogenase (LDH) concentration (C) Triple phase bone scan (D) Pulmonary function tests (PFTs) (E) Plain radiographs dedicated specifically to the ribs

C - The patient in the vignette has a history and physical exam strongly suggestive of a rib stress fracture. This is an overuse injury that is not uncommon among golfers and elite rowers due to the frequent, forceful contraction of the serratus anterior and abdominal oblique muscles involved in these activities. The diagnosis is frequently evident on plain films if obtained during the later phases of healing as callous formation occurs. Dedicated rib films despite normal chest radiographs can be helpful if a high clinical suspicion of an acute rib fracture exists after a traumatic injury, but would most likely not be diagnostic for the patient in the vignette with a stress fracture (E). The best diagnostic test to confirm a rib stress fracture is a triple phase bone scan, which would demonstrate increased uptake at the location of the injury. Treatment involves avoidance of the inciting activity for 4 to 8 weeks, followed by gradual progression back to full participation with technique modification if warranted. Computed tomography of the chest is frequently ordered in trauma settings to assess not only acute bony injury, but involvement of other structures within the chest after blunt trauma. It would not be the optimal study for the patient in the vignette given the atraumatic presentation and low probability of parenchymal involvement (A). Serum lactate dehydrogenase may be elevated in malignant chest wall pain, which is unlikely in this case given the clinical history and lack of constitutional symptoms (B). Similarly, PFTs would not likely be helpful as the history and physical exam do not suggest pulmonary pathology (D).

A 26-year-old male marathon runner presents with 3 week history of progressive fatigue and exercise intolerance after a flu like illness. His evaluation leads you to make the diagnosis of viral myocarditis. According to the 36th Bethesda guidlines you should recommend that he withdraw from competive sports and undergo a prudent convasecent period of approximately? (A) 1 month (B) 3 months (C) 6 months (D) The athlete should never return to competitive sports

C - There are currently no clinically accurate predictors of sudden death risk to guide return to play in athletes with myocarditis. The Bethesda guidelines are based on consensus/expert opinion. The Bethesda Guidelines further recommend athletes may return to training and competition after this 6 month period of time if: a LV function, wall motion, and cardiac dimensions return to normal (based on echocardiographic and/or radionuclide studies at rest and with exercise) b clinically relevant arrhythmias such as frequent and/or complex repetitive forms of ventricular or supraventricular ectopic activity are absent on ambulatory Holter monitoring and graded exercise testing c serum markers of inflammation and heart failure have normalized d the 12-lead ECG has normalized. Persistence of relatively minor ECG alterations such as some ST-T changes are not, per se, the basis for restriction from competition

A 14-year-old male presents with a subungual hematoma involving less than 25% of the nail on his right index finger after having his finger stepped on during football practice. The nail is intact. Sensation and range of motion of the index finger is intact. What is the correct statement regarding the treatment of the patient? (A) Triphenation is contraindicated (B) The patient needs a surgical consult for wound and fracture irrigation and IV antibiotics. (C) Antibiotics are not required (D) The nail must be removed and repair of nail bed performed

C - There is no difference in infection rates in subungual hematomas with an intact nail and nail margins treated with antibiotics vs. standard wound management. There is also no difference in infection rates with or without triphenation or whether or not a fracture is present. Nail bed repair is not indicated with a subungual hematoma when the nail and nail margins are intact.

A twenty-year-old star quarterback is playing in his last collegiate game, and suffers a blow to the head on the final drive before half-time. In the locker room during the break you observe him vomiting before re-joining the huddle. You pull him aside and he rpoert having a headache and begins crying about the last half of football he'll never play. His GCS is 15 and neurological exam is benign. He becomes very irritable during your exam, desperately wanting to rejoin his team. Which of the following is your best next step in management of this situation? (A) Perform serial evaluations but allow cautious return to play (B) Call the parents down from the stands and discuss return to play with them (C) Do not allow return to play and take his helmet (D) Arrange for EMS transport and suggest CT scan once in the emergency department

C - This patient has likely suffered a concussion, and should not be allowed to return to play under any circumstances. D is incorrect as neuroimaging is not helpful in the acute diagnosis or management of concussions. Serial evaluations should be performed to rule out more imminent neurological sequelae, but the athlete should not be allowed to return to play. Transparent communication with parents is important when managing concussions, but in this case return to play is not up for discussion.

Which of the following statements is true regarding sickle cell trait (SCT) and sickle cell disease (SCD)? (A) Individuals with sickle cell trait have a decreased exercise capacity (B) Individuals with sickle cell trait are not at increased risk of sudden cardiac death in the absence of other health problems (C) The presence of hemoglobin F on electrophoresis is pathognemonic of having sickle cell disease (D) Individuals from many ethnic backgrounds may have sickle cell trait

D Individuals with SCT have no significant decrease in exercise capacity. However, they are at a 28 times increased risk of sudden cardiac death. Hemoglobin F (HbF) makes up the main component of hemoglobin in fetal life; it is found as a minor component in normal adults. HbF is found in variable amounts (typically slightly elevated ~5%) in individuals with SCD. Sickle cell disease (SCD) affects millions of people throughout the world and is especially common among those whose ancestors came from sub-Saharan Africa; South America, the Caribbean, and Central America; Saudi Arabia; India; and Mediterranean countries. It is caused by a substitution of a glycine to valine at position 6 that results in reduced solubility.

A 30-year-old female presents to your clinic. She plays recreational softball and fell on her elbow two weeks ago. There was a small laceration over the elbow, but she has been trying to keep it covered with gauze. Over the last two days, the elbow has become swollen, red, and painful. She can no longer move it without severe pain. She developed a fever last night and feels nausea today. Joint aspiration confirms septic arthritis. What is true about the treatment of septic arthritis? (A) Intra-articular antibiotic treatment is the gold standard and should be given immediately (B) Septic arthritis can safely be treated with oral antibiotics as the first line of treatment, as long as the patient does not have a fever (C) Synovial fluid aspiration is rarely helpful in the treatment of septic arthritis. Blood cultures are sufficient (D) Vancomycin is a good choice for initial antibiotic treatment with septic arthritis. Antibiotic choice can then be modified based on gram stain and culture

D - A is incorrect because intra-articular antibiotics do not play a role in the treatment of septic arthritis. IV antibiotics are the gold standard. B is incorrect because IV antibiotics are the first line of treatment. Fever is not always present with septic arthritis. C is incorrect because synovial fluid aspiration is extremely important in the treatment of septic arthritis. Drainage of the joint is important, but it can also guide your antibiotic choice. D is correct because vancomycin 15-20 mg/kg/dose every eight to 12 hours with each dose not to exceed two grams is a good starting antibiotic, prior to receiving gram stain and culture results.

Which of these is true regarding causation of a chillblain, or pernio? (A) It is always associated with freezing of the tissue (B) Occurs during cold, dry conditions (C) Time to exposure is typically measured in minutes to hours (D) It is associated with an inflammatory response

D - Cold exposure injuries can be characterized as either systemic or peripheral injuries. Systemic injuries include varying degrees of hypothermia. Peripheral injuries include chillblain (pernio), frostnip, frostbite, and immersion (trench) foot. Peripheral injuries can be further characterized as either freezing or nonfreezing injuries. Both chillblain and immersion foot would be categorized in the latter, while frostbite and frostnip as freezing. A chillblain is an injury associated with an inflammatory response, making D (4) correct. Prolonged skin vasoconstriction leads to hypoxemia and vessel wall inflammation. Dermal edema may also be present. It can be characterized as a chronic, recurrent vasculitis. A is incorrect because a chillblain may or may not be associated with tissue freezing. Hands and feet are most commonly affected. B is incorrect because chillblains occurs during cold, wet conditions. C is incorrect as time to exposure is typically measured in hours to days. Frostbite, however, is measured in minutes to hours

In addition to diet and lifestyle modifications, which would be the appropriate management for an Olympic Biathlete with stage 2 hypertension? (A) Metoprolol (B) Chlorthalidone (C) Furosemide (D) Lisinopril

D - Correct answer is D. Lisinopril, an angiotensin-converting enzyme inhibitor, is the best option. Chlorthalidone and Furosemide are both diuretic medications, and are prohibited by WADA as potential masking agents for performance enhancing drugs. Metoprolol is a beta-blocker medication, and as such, is prohibited both in and out of competition in certain sports including biathlon (shooting), and archery.

A 17-year-old male American Football player is tackled, suffering an impact at the lateral part his right knee. You evaluate him on the sideline. He has tenderness to palpation at the medial joint line and valgus test is positive for increased laxity and pain. You make a clinical diagnosis and gave the athletic trainer the appropriate instructions for non-operative management and instructions for follow-up. Ten weeks later, when they present for follow up, radiographs performed by PCP reveals an abnormality next to the medial femoral condyle. Which of the following is the most likely finding? (A) Segond Fracture (B) Maisonneuve fracture (C) Fabella sign (D) Pellegrini-Stieda sign

D - Correct answer is D. The Pellegrini-Stieda sign is a finding seen on x-rays of the knee. This is a calcium deposit seen on the medial side of the knee, where the MCL attaches to the femur and is often seen in people who have had a medial collateral ligament injury. The Segond fracture is an avulsion fracture of the proximal lateral tibia just distal to the tibial plateau (A). Studies have reported an association between Segond fracture and tears of the anterior cruciate ligament (75 - 100%) and meniscii (66 - 75%). A Maisonneuve fracture is a fracture of the proximal third of the fibula associated with a tear of the distal tibiofibular syndesmosis and the interosseous membrane (B). This can be seen after an ankle inversion injury. The fabella sign is displacement of the fabella that is seen in cases of synovial effusion and popliteal fossa masses (C). The fabella is a sesamoid bone located inside the gastrocnemius lateral head tendon on the posterior side of the knee, present in about 25% of people. It can thus serve as a surrogate radio-opaque marker of the posterior border of the knee's synovium.

Midway through the season, a player complains of pain, stiffness and soreness in his groin and pubic areas following a hockey game. He notes that any coughing, sneezing, or activity causes the pain. Pain is vague and poorly localized. You evaluate him and are able to reproduce his symptoms by having him perform sit-ups and with him in a "frog position". Concerning this diagnosis, which of the following statements is correct? (A) The most common injuries involve the external and internal oblique muscle attachments and the associated aponeurosis (B) It is a common cause of acute groin pain in athletes (C) Nonoperative and operative interventions result in a pain-free return of full activities in a majority of cases (D) This is a result of an injury to the myotendinous structures adjacent to the symphysis pubis that stabilize the anterior pelvis

D - D is the correct answer. Athletic pubalgia or sports hernia occurs with weakening of the rectus abdominis, pyramidalis, internal and external obliques, transversus abdominis muscles, and/or the tendons, but no palpable hernia. The thin transversalis fascia that forms part of the posterior wall is injured. Athletic pubalgia is classically described as groin pain that localizes to the region of the pubic symphysis, with point tenderness on examination over the superficial inguinal ring without findings of an inguinal hernia. There is generally pain with twisting and turning during single-limb stance. This pain usually radiates to the adductor muscle origin region and to the testicles, often difficult for the patient to pinpoint. There may be excessive anterior pelvic tilting and/or an internal rotation of the ilium on the symptomatic side concomitant with the adductor pathology. Following sports activity, the athlete may be stiff and sore, and after competition, mobility and practice can be difficult. Physical exertion that increases intra-abdominal pressure, such as coughing or sneezing, can cause pain. The MRI image may show a tear of the hip adductors at the aponeurosis and of the rectus abdominis aponeurosis. It is a common cause of chronic groin pain in athletes (most often, soccer and ice hockey players) nonresponsive to treatment and is largely a clinical diagnosis of exclusion. Non-operative treatment does not often result in resolution of symptoms (C). However, surgical intervention may provide a pain-free return of full activities.

A 20-year-old male basketball player takes a blow to the left temporal area while coming down from making a rebound. He comes out of the game and begins complaining of seeing shadows and floaters in his left eye which progress to flashes of light over the next 10 minutes. He has some swelling over the left temple and decreased vision, but the rest of the external eye exam, including the anterior chamber, is normal. You do not have the ability to perform a funduscopic exam. His past medical history is negative, except for myopia. What is next most appropriate course of action? (A) If you rule out an orbital fracture, he can return to play (B) The player may not return to play, but he needs to see an optometrist tomorrow (C) The player may not return to play, but he needs to see an ophthalmologist tomorrow (D) He should be taken to the Emergency Department immediately. (E) The player should wear sport goggles, finish the game, and see the team ophthalmologist after the game

D - D is the correct answer. The player has suffered a trauma and has symptoms consistent with a retinal detachment. His myopia is a risk factor. With or without an orbital fracture or sport goggles, he should not return to play. He requires urgent evaluation by an ophthalmologist, who can surgically correct a retinal detachment if deemed necessary. Options A, B, C and E only delay necessary urgent evaluation and may result in permanent vision loss.

A 19-year-old soccer player presents to you with blurry vision shortly after being struck in the face with the soccer ball. What is true about the evaluation, treatment and complications of traumatic hyphema? (A) Associated with afferent pupillary defect (B) Treatment consists of doing ocular exercises in the supine position (C) May require treatment with aspirin (D) Complications include re-bleeding (E) Rarely requires specific management

D - Hyphema is the presence of blood in the anterior chamber of the eye. The eye may re-bleed after the initial traumatic hyphema, especially in cases where there is a severe myopia (nearsightedness; a condition of the eye where the light that comes in does not directly focus on the retina but in front of it, causing the image that patient sees when looking at a distant object to be out of focus). The usual treatment of traumatic hyphemas includes: analgesics, anti-emetics, head elevation, restriction of eye movement, and avoidance of therapies that may cause re-bleeding (aspirin). Ophthalmologic evaluation should be emergently sought in order to determine if management with topical steroids or operative drainage is necessary. Usually on physical exam, the patient does not demonstrate an afferent pupillary defect.

A 20-year-old running back presents to the training room for evaluation of foot pain. Earlier at practice, he was tackled from behind where he describes a rotational injury while his foot was plantar flexed. He is unable to bear weight on the foot and has edema in the midfoot region. He was sent for x-rays which show a 5 mm diastasis between the first and second metatarsal and positive fleck sign. Which of the following is the best diagnosis? (A) Navicular fracture (B) Metatarsal fracture (C) Midfoot sprain (D) Lisfranc fracture

D - Lisfranc fracture and dislocations are a rare but possible devastating injury. They require prompt diagnosis and treatment. The Lisfranc joint is a joint complex that connects the forefoot and the midfoot. The joint consists of the articulations of the first and second metatarsals with the medial and middle cuneiform bones. The Lisfranc ligament extends from the medial cuneiform to the medial aspect of the 2nd metatarsal. Typical features of the injury include midfoot swelling and inability to bear weight. Ecchymosis may develop on the plantar surface of the injured foot. Midfoot rotation or abduction will illicit pain. Disruption of the ligament creates a widening between the first and second metatarsals. A diastasis of 2 mm or more is enough to suspect the diagnosis of Lisfranc sprain. The fleck sign is avulsed bone that signifies disruption of the joint and ligamentous injury and can help differentiate from a sprain. The other two fractures (A and B) should not create a widening of the space between the first and second metatarsal.

A 16-year-old female soccer player presents to your clinic with a chief complaint of shortness of breath with wheezing and coughing during activity, particularly when she is running hard. The symptoms resolve when she stops and rests for 30 seconds to a minute. She can then continue for a short time, and then symptoms will re-occur. She is able to complete the game in this fashion with multiple rest periods to catch her breath. Her physical exam in the office is normal, pulmonary exam is clear with inspiration and expiration with 99% oxygen saturation on room air. When you put her on a treadmill, she develops shortness of breath and inspiratory stridor several minutes into the run. Her oxygen saturation remains 97-99% throughout the test. Upon cessation of the test, her symptoms resolve without intervention within 1 minute. What is your diagnosis? (A) Asthma (B) Exercise-Induced Bronchoconstriction (C) Exercise-Induced Anaphylaxis (D) Paradoxical Vocal Cord Dysfunction

D - Paradoxical Vocal Cord Dysfunction is the inappropriate movement of the vocal cords resulting in a functional airway obstruction. It affects females more than males and often mimics asthma. The findings with vocal cord dysfunction are often inspiratory with stridor (often mistaken for wheezing), cough, throat tightness, and resolution of symptoms within 5 minutes with rest. It is diagnosed clinically in many cases and is highly associated with psychosocial disorders such as depression, stress, and anxiety. Diagnosis confirmation is typically done with laryngoscopy and direct visualization of the paradoxical reaction of the vocal cords. Pulmonary function testing may also be used for diagnosis, showing a flattening of the inspiratory loop and an increased ratio of forced expiratory flow to forced inspiratory flow at 50% vital capacity, indicating extrathoracic airway obstruction. Exercise-induced bronchoconstriction and asthma both typically have expiratory wheezing as well as a sensation of chest tightness rather than throat tightness. Symptoms with both exercise-induced bronchoconstriction and asthma typically take a longer period of time to resolve, usually 15 minutes up to an hour with rest, therefore both A and B are incorrect. Both asthma and exercise-induced bronchoconstriction also resolve with the addition of albuterol, whereas vocal cord dysfunction will not respond to albuterol. Answer C is incorrect because exercise-induced anaphylaxis is characterized by signs and symptoms of anaphylaxis during physical exertion. Symptoms usually include diffuse warmth and flushing, pruritus, urticaria, and sudden fatigue.

Which of the following statements is true regarding nail disorders in athletes? (A) The risk of ingrown toenails can be minimized by having the athlete wear shoes that are snug and minimize the sliding of the foot inside the shoe (B) Trimming the toenails in a curved arc just distal to the free edge will minimize the risk of ingrown toenails (C) Any collections of dark fluid beneath the nail bed should be immediately drained with a red-hot paper clip (D) The persistence of a linear black band or streak running the length of the nail warrants further evaluation (E) The treatment of choice for onychomycosis is a topical antifungal

D - The best answer is D. The persistence of a linear black band or streak running the length of the nail may represent a melanocytic nevus or malignant melanoma of the proximal melanoma and warrants further evaluation. The risk reoccurring ingrown toenails and subungual hematoma can minimized by having the athlete wear shoes that are at least 2 cm longer then the longest toe. Cutting the toenail straight across and with enough length to clear the nail bed minimizes the risk of ingrown toenails. While subungual hematomas may be drained by pressing a red, hot paper clip end (as well as drilling with an 18 gauge needle or the use of an electrocautery unit) there is no need to do so, unless the patient is having symptoms. Asymptomatic onychomycosis does not need to be treated. However if treatment is chosen, oral antifungals are the treatment of choice.

Question Text: A 21-year-old elite male athlete plans on taking a hiking trip to the French Alps during his next vacation, and he visits you 2 weeks prior to his trip for preventative information on altitude sickness. Which of the following is true regarding acute mountain sickness (AMS)? (A) Acute mountain sickness often occurs at an altitude of 2,500 feet (B) High level of physical fitness can prevent the occurrence of AMS (C) Acetazolamide has not been proven to help prevent AMS, but it is often used off label for its ergogenic effects (D) Oxygen therapy and descent of at least 2,000 feet is an effective way to treat AMS (E) Dexamethasone is often used to treat high altitude cerebral edema (HACE), but its efficacy has yet to be proven in the treatment of AMS

D - The correct answer is D - Acute mountain sickness (AMS) typically occurs at altitudes above 1,500 meters or 4,921 feet. Being physically fit does not affect the incidence of AMS, but it can improve exercise tolerance at high altitudes. Acetazolamide reduces the incidence of acute altitude sickness, and it is approved for the treatment and prevention of AMS. Treatment of AMS, and more specifically high altitude pulmonary edema (HAPE) includes oxygen therapy (4 to 6 liters per minute), and descent of at lease 2,000 feet until symptoms resolve. Dexamethasone is indicated in treating high-altitude cerebral edema (HACE), and it has been proven to reduce the incidence of AMS.

Which of the following statements is true regarding the secondary ossification centers within the pediatric elbow joint? (A) There are a total of five secondary ossification centers within the pediatric elbow joint (B) Secondary ossification centers within the pediatric elbow joint usually appear earlier in boys than in girls (C) Secondary ossification centers within the pediatric elbow joint usually fuse earlier in boys than in girls (D) CRITOE is a useful mneumonic to help remember the order in which the secondary ossification centers within the pediatric elbow joint appear

D - The correct answer is D. CRITOE is a common and useful pneumonic to remember the order in which the secondary ossification centers within the pediatric elbow joint appear. The letters stand for capitellum, radial head, inner (medial) epicondyle, trochlea, olecranon, and external (lateral) epicondyle. The ossification center of the capitellum usually appears at around age 5 months while the ossification center of the lateral epicondyle does not appear until approximately 11 or 12 years of age. There is significant variability however in the age at which these appear. Secondary ossification centers within the pediatric elbow joint usually appear and fuse earlier in girls than in boys, making answers b and c incorrect. Answer a is incorrect as there are six, rather than five, secondary ossification centers within the pediatric elbow joint.

A 27-year-old otherwise healthy runner presents with complaints of pain in his calf while running his hill workouts, but not usually present on the flats. Pain does not consistently come on after a specific time or distance. He complains of a cramping sensation in his posterior calf, with occasional paresthesias in the sole of his foot. Symptoms generally resolve within 5-10 minutes of stopping exercise. Posterior tibialis pulse is normal in a neutral position, but decreased with active ankle plantar flexion when the knee is extended. Examination is otherwise unremarkable. Post exercise ABI (ankle brachial index) is 0.6 on the symptomatic side, 0.95 on the asymptomatic side. What would you do next for continued evaluation of this issue? (A) Compartment pressure testing pre and post exercise (B) Electrodiagnostic testing (C) Duplex ultrasonography (D) MRA of the knee with the foot in neutral and plantarflexed (E) MRI of the lumbar spine

D - The correct answer is D. This patient is describing unilateral exertional leg pain. Musculoskeletal, neurologic and vascular causes are on the differential for this presentation. This patient's abnormal ABIs indicate a vascular cause, rather than a neurologic cause. His young age makes him low risk for peripheral vascular disease causing claudication. Normal ABI is generally considered to be between 0.9-1.2. Values less than 0.66 are highly sensitive and specific for a vascular etiology. B and E are therefore incorrect. Electrodiagnostic testing would be used to evaluate for a peripheral nerve entrapment causing his symptoms or a true lumbar radiculopathy. It can also sometimes be positive in patients with spinal stenosis. Answer A is incorrect as this is generally used to diagnose chronic exertional compartment syndrome. Some practitioners are moving towards MRI to evaluate for CECS given its non invasive nature. Approximately half of the time CECS affects the anterior compartment, which will lead to symptoms referable to the deep peroneal (fibular) nerve, with paresthesias on the dorsum of the foot (particularly the first web space) and sometimes weakness of ankle dorsiflexion. 85-95% of cases of CECS are bilateral. Answer B is incorrect as this is used to diagnose external iliac artery endofibrosis. This is typically seen in athletes with repetitive hip flexion, particularly cyclists, and symptoms will typically be in three or more compartments, including the thigh. A bruit may sometimes be heard over the anterior hip at rest. This patient most likely has popliteal artery entrapment syndrome given his unilateral symptoms in the posterior calf worsened on hills or sprints, abnormal ABI, abnormal pulse in provocative positioning. Conventional angiography is still considered to be the gold standard for diagnosis of PAES, but CT angiography and MRA are increasingly being used in practice. Both can be performed in provocative positions as well. MRA can provide better detail of surrounding soft tissue and help differentiate between intrinsic vascular disease and external compression. There are five types of PAES and one venous type, depending on the anatomy of the popliteal fossa. The most common variant involves an accessory medial head of the gastrocnemius passing posterior to the popliteal artery. PAES is typically treated surgically, particularly if there are signs of ischemia.

A 15-year-old female gymnast presents to you with knee pain and swelling. She states when she bends her knee she feels a pain "inside her knee cap" and notes that she occassionally feels a popping sensation in the same area when she bends her knee. The knee seems to swell off and on, particularly when she feels the "popping". She is not aware of a specific injury. On exam, she has a mild effusion and reproducible tenderness medial to her superior patella. The most likely diagnosis is? (A) Iliotibial band syndrome (B) Patellofemoral syndrome (C) Quadriceps strain (D) Plica syndrome

D - The correct answer is plica syndrome. Plica are synovial folds or redundancy that may create mechanical symptoms. They may be present suprapatellar, infrapatellar, medial or lateral. Medial plica are most commonly symptomatic. Though a very common cause of anterior knee pain in adolescents, patellofemoral syndrome is less likely to cause mechanical symptoms or recurrent effusions. A quadriceps strain would usually be associated with an injury and not cause mechanical symptoms or effusions. The iliotibial band is on the lateral side of the knee and typically does not cause swelling or effusion.

Upon obtaining a history from the athlete, you learn that the rash first occurred four weeks ago on his chest and back while conditioning. Since that time the rash has recurred with each bout of exercise. The intensity of the pruritus has remained the same and the area of the body covered in the rash has increased with each bout of exercise. The athlete denies any symptoms of shortness of breath, dyspnea, dysphagia, chest tightness, nausea or headache associated with the occurrence of the rash. Which of the following management options is most appropriate at this time? (A) Diagnose the athlete with a psychogenic stress reaction and discuss institution of a low dose SSRI (B) Diagnose the athlete with anaphylaxis and call EMS to transport the athlete to the nearest emergency room (C) Diagnose the athlete with eczema and prescribe an over the counter skin moisturizer with prn over the counter hydrocortisone cream (D) Diagnose the athlete with urticaria (exercise-induced) and prescribe an oral antihistamine

D - The history given is consistent with exercise-induced urticaria. Classically exercise-induced urticaria initially presents as a pruritic erythematous rash of the thorax and back, however, lesions can progress to cover the entire body. An urticarial rash may be seen in individuals with exercise-induced anaphylaxis. These individuals will typically experience associated symptoms that may include a choking sensation, colic, nausea, headache, and wheezing. Ultimately, symptoms may progress to vascular collapse. The athletes history of a stable presentation is reassuring that anaphylaxis is less likely, however, the athlete and athletic training staff must be educated appropriately about the small but real risk of progression. Appropriate emergency action plans should be in place and immediate availability of epinephrine should be considered. Although a psychogenic diagnosis should always be included in the differential of any presentation, they should generally be considered a diagnosis of exclusion. The athlete does not currently exhibit signs or symptoms of anaphylaxis. An eczematous rash would lack the predictable history and presentation described.

12-year-old male presents to your office for a follow up visit after being seen one day prior in the urgent care. He sustained a right knee injury while playing with his friends. A/P plain films in the urgent care demonstrate irregularities at the articular surface of the lateral aspect of the medial femoral condyle. Exam is consistent with xray findings. An MRI shows articular cartilage thickening and low signal changes without clear margins of bone fragment. What is the next appropriate step in management? (A) Referral to orthopedic surgeon for surgical intervention (B) Immediate passive and active range of motion therapy to prevent muscle atrophy and loss of joint motion (C) Corticosteroid injection followed by return to full activity in one week (D) Knee immobilization with activity restriction

D - is correct. Initial knee immobilization and activity restriction is the initial management for all stable osteochondral lesions. Although there is much debate regarding type of immobilization, duration, and type of surgical interventions, there is consensus agreement and case series studies for use of non-operative management in stable OCD lesions. Case series show healing rates anywhere from 50-90 percent in skeletally immature (juvenile) OCD lesions. A is incorrect because stable OCD lesions in Juveniles should initially be managed non-operatively. B is incorrect because active/passive ROM therapy should not be initiated immediately but when the patient is pain free or shows radiographic signs of healing. C is incorrect because corticosteroid steroid injection should be avoided. Injection may decrease acute pain but will likely delay healing and increase progression of OCD lesion as the athlete returns to activity too quickly.

You are the team physician for a rural high school covering a Friday night football game. During the second quarter, one of the opposing team's players spears one of your players. The opposing team's player lays supine and motionless on the ground after the hit. After your initial assessment, he is alert, oriented, and has no airway compromise, but states he can't feel his arms or legs. He is spine boarded and you accompany him via ambulance to the closest hospital. En route, he begins to get feeling and movement back in his arms and legs. In the ER, the on call physician asks for your help in removing the players helmet and pads. How should you remove his equipment? (A) The helmet should be removed with the face mask on (B) Men's lacrosse and football shoulder pads and helmet create the same amount of neck flexion when the athlete is supine (C) The jersey does not need to be cut off prior to removing the shoulder pads (D) The helmet and shoulder pads can be removed only after a second person stabilizes the cspine from a caudal position

D - personnel have access to the airway if needed (Answer A is incorrect). When lying supine, the smaller shoulder pads worn by lacrosse players will place the neck in flexion, whereas football equipment are designed to keep the neck in a neutral position when the athlete is supine (Answer B is incorrect). In order for the shoulder pads to be removed, the straps and laces have to be cut. To gain easier access to the straps and laces, it is easiest to cut off the jersey. In addition, the jersey should be removed to examine the remainder of the spine for step offs or additional injuries (Answer C is incorrect). The person at the head of the bed will have control of the C-spine until the equipment is removed. Shoulder pads and helmet should be removed at the same time, but only after in line stabilization is in place from a caudal position. A cervical spine collar should also be placed at that time(Answer D is correct).

A 20-year-old male weight lifter is referred to you for follow up on a back injury. He experienced back pain following a lifting session. The pain has since resolved, but he has a plain film which shows a grade I spondylolisthesis at L5. Your physical examine reveals no pain with motion and a normal neurological examination. What is the best next step in his care? (A) Refer the athlete for spinal fusion (B) Refer the athlete for an MRI of the lumbosacral spine (C) Allow the athlete to return to non-contact sports only. Recommend that he is followed yearly for a lateral lumbar spine x-ray (D) Allow the athlete to return to all sports with out restrictions. Recommend he follow up if he has a return of pain or develops neurological symptoms (E) Place the athlete in a Boston brace and allow him to return to activity after a course of physical therapy

D - spondylolisthesis is unlikely to progress, making repeat films in the asymptomatic athlete unnecessary, especially as he has reached skeletal maturity (C). In skeletally immature patients with acute pars fracture leading to spondylolisthesis, follow up films may be recommended. Physical activity and physical therapy has not been shown to have any effect on progression or regression of spondylolisthesis. Likewise, bracing has not been shown to have any effect on spondylolisthesis (E).D is the correct answer. While reduction and fusion is often necessary for Grade IV & V spondylolisthesis, it is rarely necessary in Grade I or II (A). An MRI is not necessary in an asymptomatic athlete, but may be helpful if the athlete is experiencing neurological symptoms, which this athlete is not (B). Grade I

You are evaluating an obese 13-year-old African American male who is otherwise healthy for a three day history of right sided groin pain. He denies any recent injury. He denies fevers or rashes. After obtaining imaging, you diagnose a slipped capital femoral epiphysis (SCFE). Which of the following is true of this diagnosis? (A) A slipped capital femoral epiphysis results from a Salter Harris Type II fracture (B) The incidence rate is higher in girls than boys (C) About 80% of patients with a slipped capital femoral epiphysis will go on to have a contralateral SCFE (D) Observation is best as soon as the diagnosis is made (E) The fracture is considered stable if the patient can bear weight

E - A slipped capital femoral epiphysis results from a Salter Harris type I physeal fracture. The incidence rate in boys (13.35 per 100,000) is higher than in girls (8.07 per 100,000). Incidence is also higher among African Americans and Hispanics than Caucasians. Roughly 20% of patients have bilateral involvement at the time of presentation and an additional 20-40% will subsequently progress to bilateral slips, but prophylactic treatment of the contralateral hip is not yet recommended in most individuals. A delay in diagnosis or a period of observation can be very detrimental to the patient's outcome. Delaying treatment can lead to progression of the slip and increased severity of the SCFE which can lead to early degenerative changes. Stable slips may become unstable, which leads to higher rates of AVN. The treatment of choice therefore is immediate orthopedic referral for urgent in situ internal fixation using a single cannulated screw. Patients should be made non-weight bearing once the diagnosis is made. A SCFE is considered stable if a patient is able to bear weight and unstable if the patient cannot bear weight. Unstable SCFE is a more severe injury with a poorer prognosis though treatment goals are the same.

During a hockey game, a 7-year-old male has a primary tooth knocked out. The tooth is not fractured. What should you do immediately with the tooth before he sees the dentist? (A) Clean it well with tap water and then replant it (B) Put it in milk with urgent trasport to the dentist (C) Put it in saline solution before transport to dentist (D) Put it inside his mouth next to his buccal mucosa and go to the dentist (E) Stop the bleeding and then do nothing with the tooth

E - Answer E is the correct answer. Primary teeth should not be replanted (A). The child should still see a dentist to be sure there is not another underlying injury. Tap water can destroy the root of permanent teeth and should not be used for storage. D is incorrect as it can lead to aspiration or further injury to the tooth. B would be the preferred answer for a permanent tooth, though C may also be acceptable if milk is not available.

You are evaluating a 21-year-old women who is a member of the women's cross-country team for menstrual dysfunction. She reports that she has had irregular periods since menarche at age 15. She typically has 3-6 periods per year, her last period was about 2.5 months ago. Her other past medical history and review of systems is only remarkable for a history of a metatarsal stress fracture her junior year of high school. The most important initial test(s) will be? (A) Progesterone challenge with Medroxyprogesterone acetate (Provera/Cycrin) 10 mg each day for five days (B) Progesterone challenge with Progesterone in oil 200 mg IM. (C) Follicle stimulating hormone (FSH), luteinizing hormone (LH), and prolactin tests (D) Thyroid stimulating test (TSH) (E) Pregnancy test

E - While FSH, and prolactin testing are important in the work-up of athletic amenorrhea, the most important first test is a pregnancy test. Other tests that are routinely measured include a TSH. Luteinizing hormone and estrogen levels are rarely helpful. The progesterone challenge is useful in assessing the amount of endogenous estrogens and the outflow tract anatomy.

A 40-year-old male presents to your office after injuring his right lower leg playing tennis. He states that as he ran back to hit an overhead he felt a tearing sensation in his lower calf. He has had difficulty walking since, and has noticed his right calf is much more swollen than his left. Following your regarding this injury? (A) This injury is commonly seen in young athletes and is associated with limited disability and symptoms (B) This injury commonly involves the lateral head of the gastrocnemius at the musculotendinous junction (C) The mechanism of injury is plantarflexion of the ankle combined with knee flexion (D) Surgical repair is often required to restore function following this injury (E) Acute compartment syndrome of the superficial posterior compartment is a complication of this injury due to significant swelling

E - jury seen in middle aged athletes. This injury involves a tear or rupture of the medial head of the gastrocnemius muscle at its musculotendinous junction. This injury is caused by ankle dorsiflexion combined with knee hyperextension. This injury is associated with significant pain, swelling, ecchymosis and difficulty walking. If swelling is severe, acute compartment syndrome of the superficial posterior compartment of the lower leg can develop and this is a surgical treated with conservative management including crutches if necessary, activity modification, ice, compression, anti-inflammatory medication and physical therapy.

During a triathlon a participant falls from his bicycle and sustains a fracture of his left scapula. You see him in the office 4 weeks after the incident with persistent complaints of weakness of the left upper extremity and vague shoulder pain. On physical examination he has relative weakness and pain with both abduction and external rotation of the left glenohumeral joint when compared to the right. Forward flexion and internal rotation are within normal limits as is light touch sensation in both upper extremities. A non contrast MRI of the left shoulder reveals no abnormalities and healing of the scapula fracture. What is the most likely diagnosis ? (A) Parsonage-Turner Syndrome (B) Axillary nerve injury (C) Suprascapular nerve injury (D) Long thoracic nerve injury

This patient sustained an injury to his suprascapular nerve secondary to the scapula fracture. The suprascapular nerve innervates both the supraspinatus and the infraspinatus muscles and can be injured secondary to rotator cuff tears, shoulder arthrodesis, and fractures of the scapula. Symptoms and signs can include vague shoulder and/or acromioclavicular joint pain, possible radiation of the pain in the suprascapular or radial nerve distribution, and weakness in the supraspinatus and/or infraspinatus muscles, therefore affecting abduction and external rotation. Parsonage-Turner Syndrome refers to a rare set of symptoms resulting from inflammation of unknown etiology of the brachial plexus. Patients who suffer from this usually experience sudden onset pain radiating from the shoulder to the upper arm and affected muscles can become weak, atrophied, and in advanced cases, paralyzed. Axillary nerve injury can be traumatic or atraumatic and usually results in posterior shoulder pain, parasthesias over a circular location of the lateral arm, weakness of the deltoid muscle, and worsening of the pain with forward flexion and/or abduction and external rotation. B is incorrect because the patient has no sensory deficits and no noted deltoid muscle weakness. Long thoracic nerve injury can also be traumatic or atraumatic in nature and usually presents with scapular winging, pain in the shoulder, neck, and arm, and weakness in the upper extremity. The patient above has no scapular winging noted so D is incorrect.


Ensembles d'études connexes

FUNCTIONAL TYPES OF SKELETAL MUSCLE FIBERS

View Set

Module 3- AWS Global Infrastructure Overview

View Set

Test 2-take by saturday morning! testing center!

View Set

Marketing Management Chapter 1 MC

View Set

Chapter 17 (quizzes and test from heather)

View Set